Você está na página 1de 53

ANALYSIS PM20002

In this module we deal with sequences, series and functions of one real variable. The main
subjects are
Cauchy sequences;
Uniformly continuous functions;
Convergent series;
Dierentiable functions;
Riemann integral.
1
We start by recalling several concepts and fact, you have learned last year. First, recall
some notation.
Set theoretic notation: x A means that x is an element of the set A; A B means
that the set A is a subset of the set B; AB is the union of the sets A and B; AB
is the intersection of the sets A and B.
Specic sets: R is the set of real numbers;
Z = {. . . , 2, 1, 0, 1, 2, . . . } is the set of integers;
N = {1, 2, . . . } is the set of positive integers (=natural numbers);
Q is the set of rational numbers;
C is the set of complex numbers;
for a, b R, a < b, the nite intervals [a, b], (a, b), (a, b] and [a, b) are dened as usual:
[a, b] = {x R : a x b}, (a, b) = {x R : a < x < b},
[a, b) = {x R : a x < b}, (a, b] = {x R : a < x b}.
for a R, the innite intervals [a, ), (a, ), (, a] and (, a) are dened as
usual:
[a, ) = {x R : x a}, (a, ) = {x R : x > a},
(, a] = {x R : x a}, (, a) = {x R : x < a}.
For instance, (0, ) is the set of positive real numbers.
Sequences: a sequence a
1
, a
2
, . . . will be denoted {a
n
}

n=1
or {a
n
}
nN
. Note that the
index n may not necessarily start at 1. So sequences {a
n
}

n=0
or {a
n
}

n=3
are in order.
Functions: For two sets A and B of real numbers, f : A B denotes a function f
dened on A and taking values in B. That is, to each x A, f assigns the number
f(x) B.
Notation for summation: Instead of a
1
+ a
2
+ . . . + a
n
, we shall often write
n

j=1
a
j
:
a
1
+ . . . + a
n
=
n

j=1
a
j
.
For instance 1 + 2 + . . . + 100 =
100

j=1
j (= 5050).
2
We will repeatedly use the triangle inequality:
|a + b| |a| +|b| for each a, b R.
For sums of more than two numbers, the triangle inequality reads:
|a
1
+ . . . + a
n
| |a
1
| + . . . +|a
n
|
or

j=1
a
j

j=1
|a
j
|
for any n N and any a
1
, . . . , a
n
R.
As you have seen, the triangle inequality implies the following useful inequality:
|a| |b| ||a| |b|| |a b| for each a, b R.
3
0.1 Supremum and innum
Denition 0.1. A number u R is called an upper bound for a set A R if a u for each
a A. A number l R is called an lower bound for a set A R if a l for each a A.
A set A R is said to be bounded above if it has an upper bound and A is said to be
bounded below if it has a lower bound. Furthermore, A is called bounded if it is bounded
above and bounded below.
Denition 0.2. Let A R. A number s R is called the maximum of the set A if s A
and s is an upper bound for A. If s is the maximum of A, we write s = max A. A number
w R is called the minimum of the set A if w A and w is an lower bound for A. If w is
the minimum of A, we write w = min A.
Denition 0.3. Let A R be non-empty and bounded above. A number s R is called
the supremum of the set A or the least upper bound of A if
s is an upper bound for A and
for each > 0 there is a A such that a > s .
If s is a supremum of A, we write s = sup A.
Similarly
Denition 0.4. Let A R be non-empty and bounded below. A number w R is called
the innum of the set A or the greatest lower bound of A if
w is an lower bound for A and
for each > 0 there is a A such that a < w + .
If w is a innum of A, we write w = inf A.
Remark 0.5. 1. Supremum and innum are uniquely determined by the set provided they
do exist. Thus we speak of the supremum and the innum of a set.
2. If A R has a maximum c, then sup A = c. If A has a minimum w, then inf A = w.
Moreover, A has a maximum if and only if A is bounded above and sup A A. Similarly, A
has a minimum if and only if A is bounded below and inf A A.
3. We can apply the above concepts to a sequence {a
n
}
nN
by considering the set A =
{a
n
: n N}. For instance, supremum and innum of {a
n
} are (by denition) supremum
and innum of A; {a
n
} is bounded above (resp., below) i so is the set A etc.
4. We can also apply the above concepts to a function f : A B by considering the set
A = {f(x) : x A}. For instance, supremum and innum of f are (by denition) supremum
and innum of A; f is bounded above (resp., below) i so is the set A etc.
The Completeness Axiom for real numbers: Every non-empty bounded
above subset A of R has a supremum.
The above axiom (in a number of possible variations) appears in any axiomatic denition
of real numbers. Another way is to give a constructive denition of real numbers (there
are several of them) and then prove the completeness property. We are somewhere in the
middle. We neither discuss the axioms of the real line nor provide a rigorous construction,
but rather employ an intuitive approach to real numbers and single out the completeness
axiom as the one being least obvious.
4
Exercise. Prove that The Completeness Axiom for real numbers is equivalent to the
statement that every non-empty bounded below subset A of R has an innum.
Hint: consider the set A = {a : a A} and observe that inf(A) = sup A and
sup(A) = inf A.
We also mention the following easy but useful property of inf and sup.
Proposition 0.6. Let A R be bounded and B is a non-empty subset of A. Then B is also
bounded and
inf A inf B sup B sup A.
For the sake of completeness, we also recall the formula for inf and sup of the union of
two bounded non-empty sets A, B R:
sup(A B) = max{sup A, sup B}, inf(A B) = min{inf A, inf B}.
5
0.2 Convergent sequences
Denition 0.7. A sequence {x
n
}
nN
of real numbers is said to converge to s R if for each
> 0, there is n
0
N such that |x
n
s| < for any n n
0
.
If {x
n
} converges to s, we write x
n
s or lim
n
x
n
= s or limx
n
= s. The number s is
called the limit of the sequence {x
n
}.
If a sequence is not convergent, it is called divergent.
We list a number of useful properties of convergent sequences.
(C0) The limit of a convergence sequence is unique. That is, if x
n
s and x
n
s

, then
s = s

;
(C1) The constant sequence x
n
= c (for each n N) converges to c;
(C2) If x
n
s and y
n
t, then x
n
+ y
n
s + t, x
n
y
n
st, cx
n
cs for each c R and
x
n
y
n

s
t
provided t = 0 and y
n
= 0 for n N;
(C3) x
n
0 if and only if |x
n
| 0;
(C4) the sandwich theorem: if x
n
s, y
n
s and x
n
z
n
y
n
for each n N, then
z
n
s;
(C5) every convergent sequence is bounded;
(C6) a subsequence of a convergent sequences converges to the same limit. That is, if x
n
s
and {n
k
}
kN
is a strictly increasing sequence of positive integers, then the sequence
{x
n
k
}
kN
also converges to s;
(C7) if x
n
y
n
for each n N, x
n
s and y
n
t, then s t;
(C8) if {x
n
} is bounded and y
n
0, then x
n
y
n
0;
(C9) every bounded monotonic (=increasing or decreasing) sequence is convergent. More-
over, a bounded increasing sequence converges to its supremum, while a bounded
decreasing function converge to its innum.
Example 0.8. If a > 0, then lim
n
n
a
= 0. If 0 < a < 1, then lim
n
a
n
= 0.
Proof. DIY using the denition.
Example 0.9. lim
n
8n
3
+2n
2
3
4n
3
n

n
= 2.
Proof. Clearly
8n
3
+ 2n
2
3
4n
3
n

n
=
8 + 2n
1
3n
3
4 n
1/2
.
We can us Example 0.8 and (C3) to see that the numerator in the above fraction converges to
8, while the numerator converges to 4. Using (C3) again, we get lim
n
8n
3
+2n
2
3
4n
3
n

n
=
8
4
= 2.
Sometimes it is worth to consider innite limits.
6
Denition 0.10. A sequence {x
n
}
nN
of real numbers is said to converge to + if for each
c > 0, there is n
0
N such that x
n
> c for any n n
0
.
A sequence {x
n
}
nN
of real numbers is said to converge to if for each c > 0, there is
n
0
N such that x
n
< c for any n n
0
.
NOTE that if x
n
+ or x
n
, the sequence {x
n
} is NOT convergent. That is,
convergence to is a particular case of divergence.
Proposition 0.11. Let {x
n
} be a sequence of non-zero real numbers. Then x
n
0 if and
only if
1
|x
n
|
+.
Proof. First assume that x
n
0. Let c > 0. Applying the denition of the limit with
= c
1
, we see that there is n
0
N such that |x
n
| < c
1
for n n
0
. That is, |x
n
|
1
> c for
n N. By denition, |x
n
|
1
+.
Next, assume that |x
n
|
1
+. Let > 0. Applying the denition of the innite limit
with c =
1
, we see that there is n
0
N such that |x
n
|
1
>
1
for n n
0
. That is, |x
n
| <
for n n
0
. By denition, x
n
0.
7
0.3 The nested intervals theorem and the BolzanoWeierstrass
theorem
Denition 0.12. A sequence {A
n
}
nN
of sets is called nested if A
n+1
A
n
for n N.
It is worth noting that a nested sequence of non-empty open intervals can have empty
intersection. For instance if A
n
= (0, 2
n
) for n N (that is, A
1
= (0, 1/2), A
2
= (0, 1/4),
etc.) then the intersection of the sequence A
n
is empty. However, if we consider the sequence
of closed intervals with the same ends, the intersection turns out to be non-empty. Namely,
the intervals B
n
= [0, 2
n
] have the intersection being the one-element set {0}. As a matter
of fact, this happens with any nested sequence of (bounded) closed intervals. The following
theorem is actually an equivalent version of the completeness axiom. It is called the nested
intervals theorem.
Theorem 0.13. Let {I
n
} be a nested sequence of bounded closed intervals. Then the inter-
section

n=1
I
n
is non-empty. In other words, there exists a R such that a I
n
for each
n N.
A useful application of the notion of the Cauchy sequence is the following theorem, known
as the BolzanoWeierstrass Theorem.
Theorem 0.14. Any bounded sequence of real numbers has a convergent subsequence.
8
0.4 Continuous functions
Denition 0.15. If I is a nite or innite interval of the real line and f : I R, then f
is called continuous at c I if for each > 0, there is > 0 such that |f(x) f(c)| < ,
whenever x I and |x c| < .
A function f : I R is called continuous whenever f is continuous at every c I.
The following characterization of continuity is very useful.
Theorem 0.16. Let I be an interval of the real line and f : I R. Then f is continuous
at c I if and only if f(x
n
) f(c) for any sequence {x
n
}
nN
of elements of I such that
x
n
c.
You already know that the following functions are continuous:
any polynomial p : R R, p(x) = a
0
+ a
1
x + . . . + a
n
x
n
;
f : (0, ) R, f(x) = x
a
for each a R;
f : R R, f(x) = a
x
for each a > 0;
f : (0, ) R, f(x) = ln x;
f : R R, f(x) = sin x;
f : R R, f(x) = cos x.
Continuity (at a point as well as on an interval) is conserved by a number of operations.
Namely:
the sum and the product of two continuous functions is continuous;
if f and g are continuous and g does not attain the value 0, then f/g is continuous;
if I and J are intervals and f : I J is continuous and bijective, then the inverse
map f
1
: J I is continuous.
composition of two continuous functions is continuous.
We also state two useful theorems dealing with continuous functions. The next Theorem
is called the Intermediate Value Theorem.
Theorem 0.17. Let I be an interval of the real line, f : I R be a continuous function
and a, b I, a < b. Then for any R lying between f(a) and f(b) (that is either
f(a) < < f(b) or f(b) < < f(a)), there is c (a, b) such that f(c) = .
In other words, for any two values of f, f attains also all values between them.
Theorem 0.18. Let I be a closed bounded interval and f : I R be a continuous function.
Then f is bounded and f attains its maximal and minimal values. That is, there are s, t I
such that f(s) f(x) f(t) for each x I.
9
0.5 Limits of functions
Denition 0.19. Let c R and f be a real valued function dened on an interval I
containing c, except for maybe the point c. We say that t R is the limit of f at c and write
lim
xc
xI
f(x) = t if for any > 0, there exists > 0 such that |f(x) t| < whenever x I and
0 < |x c| < .
If I has the shape (c, b) with b > a and f : I R has the limit at c, it is called the limit
of f at c from the right and denoted lim
xc+
f(x) = t. In this case for any > 0, there exists
> 0 such that |f(x) t| < whenever c < x < c + .
Similarly, if I = (a, c) with a < c and f : I R has the limit at c, it is called the limit of
f at c from the left and denoted lim
xc
f(x) = t. In this case for any > 0, there exists > 0
such that |f(x) t| < whenever c < x < c.
If c is strictly inside I, the limit is usually called two-sided. In this case we simply write
lim
xc
f(x) = t.
Whether f is or is not dened at c does not matter, since the value of f at c does not
appear in the denition. We start by observing that two-sided and one-sided limits are
uniquely dened by the function.
Recall that a function f : I R (I is an interval) is called increasing (on I) if f(x) f(y)
for any x, y I satisfying x < y and f is called decreasing (on I) if f(x) f(y) for any
x, y I satisfying x < y. Similarly, f is called strictly increasing (on I) if f(x) < f(y) for
any x, y I satisfying x < y and f is called strictly decreasing (on I) if f(x) > f(y) for any
x, y I satisfying x > y.
A function f is called monotonic (sometimes monotone) if it is either increasing or de-
creasing.
For example, a constant function f(x) = c is both increasing and decreasing on R and
is neither strictly increasing nor strictly decreasing. The function f(x) = sin x is strictly
increasing on the interval [/2, /2], is strictly decreasing on the interval [/2, 3/2] and
is neither decreasing nor increasing (hence non-monotonic) on the interval [0, ].
We recall a number of properties of limits.
Uniqueness. If lim
xc
f(x) = t and lim
xc
f(x) = s, then s = t. If lim
xc+
f(x) = t and
lim
xc+
f(x) = s, then s = t. If lim
xc
f(x) = t and lim
xc
f(x) = s, then s = t.
Two sided limits versus one-sided. The two-sided limit of f at c exists and equals
t if and only if both one-sided limits of f at c exist and both are equal t.
Heines Denition. f : I R has the limit t at c if and only if f(x
n
) t for every
sequence {x
n
} in I such that x
n
c and x
n
= c for every n N.
The constant function f(x) = t for any x R has the limit t at any point.
Locality. If > 0 and two functions f and g (are dened and) coincide on (c
, c) (c, c + ), then either both f and g do not have a limit at c or they have the
same limit at c : lim
xc
f(x) = lim
xc
g(x). Similar statements are true for one-sided limits.
Boundedness. If f : I R has a limit at c, then there is > 0 such that f is
bounded on (c , c + ) I.
10
Relation to continuity. A function f : I R is continuous at c I if and only if
lim
xc
xI
f(x) = f(c).
Arithmetic properties. If f, g : I R, s = lim
xc
xI
f(x) and t = lim
xc
xI
g(x), then
s + t = lim
xc
xI
f(x) + g(x), st = lim
xc
xI
f(x)g(x), rs = lim
xc
xI
rf(x) for each r R and
s
t
= lim
xc
xI
f(x)
g(x)
provided t = 0.
Limits of compositions. Let I and J be intervals, c I, f : I J and g : J R.
Assume also that lim
xc
xI
f(x) = t J, lim
yt
yJ
g(y) = s R and at least one of the following
conditions is satised: either g is continuous at t or t / f(I \{c}). Then lim
xc
xI
(gf)(x) =
s.
Limits and inequalities. Let f, g : I R, f(x) g(x) for each x I \ {c} and
f(x) and g(x) both converge as x c. Then lim
xc
xI
f(x) lim
xc
xI
g(x).
Sandwich rule. If f, g, h : I R, f(x) h(x) g(x) for each x I \ {c} and
lim
xc
xI
f(x) = lim
xc
xI
g(x) = t R. Then lim
xc
xI
h(x) = t.
Let f : I\ R. Then lim
xc
xI
f(x) = 0 if and only if lim
xc
xI
|f(x)| = 0.
Let f, g : I R, g is bounded and lim
xc
xI
f(x) = 0. Then lim
xc
xI
f(x)g(x) = 0.
Limits of monotonic functions. If f : (c, b) R is bounded and monotonic, then
f(x) converges as x c+. If g : (a, c) R is bounded and monotonic, then g(x)
converges as x c. Thus if I is an open interval, c I and h : I R is monotonic,
then h(x) converges as x c+ and h(x) converges as x c (possibly to dierent
numbers!).
Finally, we recall the concepts of innite limits and limits at innity.
Denition 0.20. Let f : (a, ) R. We say that f(x) converges to t R as x if
for each > 0 there is C > 0 such that |f(x) t| < for x > C.
Let f : (, a) R. We say that f(x) converges to t R as x if for each > 0
there is C > 0 such that |f(x) t| < for x < C.
Denition 0.21. Let c R and f be a real valued dened on (c , c + ) \ {c}. We
say that f(x) + as x c if for each C > 0 there is > 0 such that f(x) > C for
0 < |x c| < . In this case we write lim
xc
f(x) = +.
We say that f(x) as x c if for each C > 0 there is > 0 such that f(x) < C
for 0 < |x c| < . In this case we write lim
xc
f(x) = .
The statements like lim
xc
f(x) = or lim
x
f(x) = are dened similarly. Limits at
innity enjoy all properties of the limit as the limits at a nite point. As for the sequences,
for f, which does not attain 0, |f(x)| + if and only if
1
|f(x)|
0.
11
0.6 Examples of computing limits
The properties of limits, we have so far, together with the fact that lim
xc
f(x) = f(c) if f is
continuous at c allows us to compute many specic limits. Here are some examples:
1. lim
x1
x
3
1
x
2
1
= lim
x1
(x1)(x
2
+x+1)
(x1)(x+1)
= lim
x1
x
2
+x+1
x+1
=
x
2
+x+1
x+1

x=1
=
3
2
.
We have just substituted x = 1 into
x
2
+x+1
x+1
, because this function is continuous at 1.
2. lim
x0
x
2
+x
4
x
2
+x
3
= lim
x0
1+x
2
1+x
=
1+x
2
1+x

x=0
= 1.
3. lim
x1
(x1) sin x
x
4
1
= lim
x1
(x1) sin x
(x1)(x+1)(x
2
+1)
= lim
x1
sin x
(x+1)(x
2
+1)
=
sin x
(x+1)(x
2
+1)

x=1
=
sin1
4
.
4. lim
x0
x sin(1/x) = 0 since sin(1/x) is bounded and lim
x0
x = 0.
5. lim
x0+
cos((/4)+

x sin(1/x)). Since lim


x0+

x = 0 and sin(1/x) is bounded, lim


x0+

x sin(1/x) =
0. Hence lim
x0+
((/4) +

x sin(1/x)) = /4. Since cos is continuous at /4, the limit of com-


position property implies that lim
x0+
cos((/4) +

x sin(1/x)) = cos(/4) =
1

2
.
6. Limits of the gluing of two continuous functions. Let f and g be continuous at c R
and
h(x) =
_
f(x) if x < c;
g(x) if x > c.
The function h can be undened at c or we can consider it to be taking any value we like
at c. Since f and g are continuous at c, f(c) = lim
xc
f(x) = lim
xc
f(x) and g(c) = lim
xc
g(x) =
lim
xc+
g(x). Since h(x) coincides with f(x) for x < c, lim
xc
h(x) = lim
xc
f(x) = f(c). Since
h(x) coincides with g(x) for x > c, lim
xc+
h(x) = lim
xc+
g(x) = g(c). Thus
lim
xc
h(x) = f(c) and lim
xc+
h(x) = g(c).
It follows that lim
xc
h(x) does exist if and only if f(c) = g(c).
7. It is a particular case of the above example with f(x)1, g(x) = 1 (constant functions)
and c = 0. Namely, the function sign : R R is dened by the formula
sign (x) =
_
_
_
1 if x < 0;
1 if x > 0;
0 if x = 0.
By 6. lim
x0
sign (x) = 1, lim
x0+
sign (x) = 1 and lim
x0
sign (x) does not exist.
8. Exercise. Using the denition of the limit, verify that
lim
x0+
x
a
= + for any a < 0,
lim
x0
x
1
= ,
lim
x+
a
x
= + for any a > 1,
lim
x
a
x
= 0 for any a > 1.
12
9. Consider the function f : R R,
f(x) =
_
sin(1/x) if x = 0,
0 if x = 0.
Then lim
x0+
f(x) and lim
x0
f(x) do not exist.
Indeed, let x
n
=
1
2n
and y
n
=
1
(2n+1/2)
for n N. It is easy to see that x
n
> 0, y
n
> 0,
x
n
0 and y
n
0. On the other hand, f(x
n
) = f(x
n
) = 0, f(y
n
) = 1 and f(y
n
) = 1
(check it!) for each n N. Therefore {x
n
} and {y
n
} are two sequences of positive numbers
such that the sequences {f(x
n
)} and {f(y
n
)} have dierent limits (0 and 1 respectively).
Thus lim
x0+
f(x) does not exist. Moreover {x
n
} and {y
n
} are two sequences of negative
numbers such that the sequences {f(x
n
)} and {f(y
n
)} have dierent limits (0 and 1
respectively). Thus lim
x0
f(x) does not exist.
1 Cauchy sequences
There are cases, when it is dicult or even impossible to nd the limit of a convergent
sequence, but possible to show that the sequence converges. The concept allowing us to do
that is the notion of a Cauchy sequence.
Denition 1.1. A sequence {x
n
}
nN
is called a Cauchy sequence if for each > 0 there is
n
0
N such that |x
n
x
m
| < for any m, n N, m, n n
0
.
First, we shall demonstrate that a sequence is a Cauchy sequence if and only if it converges.
Thus if we want to show that a sequence converges without specifying its limit, it suces to
verify that our sequence is a Cauchy sequence.
Lemma 1.2. Every Cauchy sequence is bounded.
Proof. Let {x
n
}
nN
be a Cauchy sequence. Applying the denition of a Cauchy sequence
with = 1, we see that there is n
0
N such that |x
m
x
n
| < 1 for m, n n
0
. Fixing
m = n
0
, we get |x
n
x
n
0
| < 1 for n n
0
. By the triangle inequality
|x
n
| |x
n
0
| +|x
n
x
n
0
| < |x
n
0
| + 1 for n n
0
.
Hence
|x
n
| max{|x
1
|, . . . , |x
n
0
1
|, |x
n
0
| + 1} for each n N.
Hence {x
n
} is bounded.
Lemma 1.3. If a Cauchy sequence {x
n
}
nN
has a convergent subsequence, then {x
n
} con-
verges.
Proof. Let {x
n
} be a Cauchy sequence, whose subsequence {x
n
k
} converges to a R. Let
> 0. Since {x
n
} is Cauchy, there is N N such that
|x
m
x
n
| < /2 whenever m, n N.
Since x
n
k
a, we can nd l N such that n
l
N and |x
n
l
a| < /2. Now let n N be
such that n N. By the above display with m = n
l
, we have |x
n
x
n
l
| < /2. Using the
last two inequalities together with the triangle inequality, we get
|x
n
a| = |(x
n
x
n
l
) + (x
n
l
a)| |x
n
x
n
l
| +|x
n
l
a| <

2
+

2
=
for each n N. By denition, {x
n
} converges to a.
13
Theorem 1.4. Let {x
n
}
nN
be a sequence of real numbers. Then {x
n
} converges if and only
if {x
n
} is a Cauchy sequence.
Proof. First assume that {x
n
} converges. That is, there is s R such that x
n
s. Pick
> 0. By denition of convergence, there is n
0
N such that |x
n
s| < /2 for n n
0
.
Thus, if m, n N and m, n n
0
, we have |x
n
s| < /2 and |x
m
s| < /2. By the triangle
inequality,
|x
m
x
n
| |x
m
s| +|x
n
s| < (/2) + (/2) = .
That is, |x
m
x
n
| < whenever m, n n
0
and therefore {x
n
} is a Cauchy sequence.
Next, we assume that {x
n
} is a Cauchy sequence. Lemma 1.2, {x
n
} is bounded. By the
BolzanoWeierstrass theorem, {x
n
} has a convergent subsequence {x
n
k
}. Thus {x
n
} is a
Cauchy sequence with a convergent subsequence. By Lemma 1.3, {x
n
} converges.
We give an example of how to verify that a sequence is a Cauchy sequence, using only the
denition. Let x
n
=
n
n+2
for n N. Then {x
n
}
nN
is a Cauchy sequence. To do this, we
have to estimate the dierences |x
n
x
m
|. Let m, n N and m > n. Then
|x
n
x
m
| =

n
n + 2

n
n + 2

2
n + 2

2
m + 2

2
n + 2
.
Now let > 0. Find n
0
N such that
2
n
0
+2
< (just take n
0
>
2

2). Then whenever


m > n n
0
, |x
m
x
n
|
2
n+2

2
n
0
+2
< . Hence {x
n
} is a Cauchy sequence.
If you suspect that a certain sequence is not convergent, then the best way to show it, is
to verify that it is not a Cauchy sequence.
Example 1.5. Let x
n
=
n(1)
n
n+2
for n N. Then {x
n
} is not convergent.
Proof. If k N, then x
2k
=
2k
2k+2
= 1
1
k+1

1
2
and x
2k1
=
2k+1
2k+1
= 1+
2
2k+1

1
3
. Thus
x
2k
x
2k1

1
2
+
1
3
=
5
6
.
Now for each n
0
N, we can pick k N such that n = 2k 1 n
0
(for example, k = n
0
will do). Then m = 2k n
0
and |x
n
x
m
| = x
2k
x
2k1

5
6
. Thus {x
n
} fails to be a
Cauchy sequence: just take =
5
6
.
We illustrate the notion of the Cauchy sequence by the following examples. For sake of
convenience, we rst provide an equivalent form of the denition of a Cauchy sequence.
Lemma 1.6. Let {x
n
}
nN
be a sequence of real numbers for which there exists a sequence
{c
n
}
nN
of positive numbers such that c
n
0 and |x
n
x
m
| c
m
whenever n > m. Then
{x
n
} is a Cauchy sequence.
Proof. Let > 0. Since c
n
0, there is n
0
N such that c
n
< for every n n
0
. Now let
m, n n
0
. If m = n, then |x
m
x
n
| = 0 < . If n > m, then |x
n
x
m
| c
m
< . Finally, if
n < m, then |x
n
x
m
| = |x
m
x
n
| c
n
< . Thus |x
n
x
m
| < whenever m, n n
0
and
therefore {x
n
} is a Cauchy sequence.
Remark 1.7. The above lemma works in the opposite direction as well. Namely, if {x
n
} is
a Cauchy sequence, then there is a sequence {c
n
} of positive numbers such that c
n
0 and
|x
n
x
m
| c
m
whenever n > m. (Try and prove it!)
14
Example 1.8. Consider the sequence {x
n
}
nN
dened by the formula
x
n
=
cos 1
3
+ cos 29 + . . . + cos n3
n
=
n

j=1
cos j3
j
.
Then {x
n
} converges.
Proof. Let n > m. Then
|x
n
x
m
| =

j=1
cos j
3
j

m

j=1
cos j
3
j

j=m+1
cos j
3
j

Since | cos j| 1, we can use the triangle inequality to see that


|x
n
x
m
|
n

j=m+1
| cos j|
3
j

n

j=m+1
1
3
j
=
1
3
m+1
nm1

j=0
1
3
j
.
Using the formula 1 + q + . . . + q
k
=
1q
k+1
1q
, which holds whenever q = 1, we obtain
|x
n
x
m
|
1 3
mn
3
m+1
(1 3
1
)

1
3
m+1
(1 3
1
)
=
1
2 3
m
.
Since
1
23
m
0, Lemma 1.6 ensures that {x
n
} is a Cauchy sequence. By Theorem 1.4, {x
n
}
converges.
Example 1.9. Consider the sequence {x
n
}
nN
dened by the formula
x
n
=
1
2
+
1
12
+ . . . +
1
(2n 1)2n
=
n

j=1
1
(2j 1)2j
.
Then {x
n
} converges.
Proof. Let n > m. Then
|x
n
x
m
| =
n

j=1
1
(2j 1)2j

m

j=1
1
(2j 1)2j
=
n

j=m+1
1
(2j 1)2j
.
Since
1
2j(2j1)
=
1
2j1

1
2j
and the latter is the length of the interval I
j
=
_
1
2j
,
1
2j1

, |x
n
x
m
|
is the sum of the lengths of the intervals I
j
for m + 1 j n. Since these intervals do not
overlap and lie in the interval J =
_
0,
1
2m+1

, their total length does not exceed the length


of J, which is
1
2m+1
:
|x
n
x
m
|
1
2m + 1
for n > m.
Since
1
2m+1
0, Lemma 1.6 ensures that {x
n
} is a Cauchy sequence. By Theorem 1.4, {x
n
}
converges.
Example 1.10. Consider the sequence {x
n
}
nN
dened by x
1
= 0 and x
n+1
= cos x
n
for
n N. Then {x
n
} converges. Note that there is no known expression of the limit s of {x
n
}
in terms of standard constants like rationals, , e etc.
15
Proof. First, we try and localize the sequence {x
n
} within a convenient interval. Clearly
x
2
= cos x
1
= cos 0 = 1 and x
3
= cos x
2
= cos 1. Thus x
2
, x
3
I = [cos 1, 1]. It is also easy
to see that the function cos restricted to I takes values in [cos(cos 1), cos 1] I. That is cos
maps I into itself. It follows (see the denition of x
n
) that x
n
I for each n 2. Next, for
any n 2, using some elementary trigonometry, we get
|x
n+2
x
n+1
| = | cos x
n+1
cos(x
n
)| = 2| sin((x
n+1
x
n
)/2)| | sin((x
n+1
+ x
n
)/2)|.
Using the inequality | sin t| |t|, we get
2| sin((x
n+1
x
n
)/2)| 2|(x
n+1
x
n
)/2| = |x
n+1
x
n
|.
Since x
n
, x
n+1
I, we have that their midpoint (x
n+1
+ x
n
)/2 is also in I. Since sin is
non-negative and increasing on I, we get
| sin((x
n+1
+ x
n
)/2)| = sin((x
n+1
+ x
n
)/2) q = sin 1 < 1.
Combining the last three displays, we obtain
|x
n+2
x
n+1
| q|x
n+1
x
n
| for any n 2.
If m > n 2, we then have
|x
m+1
x
m
| q|x
m
x
m1
| q
2
|x
m1
x
m2
| . . .
. . . q
mn1
|x
n+2
x
n+1
| q
mn
|x
n+1
x
n
|.
That is, |x
m+1
x
m
| q
mn
|x
n+1
x
n
| whenever m > n 2. Now if m > n 2, then
x
m
x
n
= x
m
x
m1
+ x
m1
x
m2
+ . . . + x
n+2
x
n+1
+ x
n+1
x
n
=
mn

j=1
x
n+j
x
n+j1
.
By the triangle inequality,
|x
m
x
n
|
mn

j=1
|x
n+j
x
n+j1
|.
Since |x
n+j
x
n+j1
| q
j1
|x
n+1
x
n
|, we get
|x
m
x
n
| |x
n+1
x
n
|(1+q +q
2
+. . . +q
mn1
) =
1 q
mn
1 q
|x
n+1
x
n
|
1
1 q
|x
n+1
x
n
|.
Since |x
n+1
x
n
| q
n2
|x
3
x
2
|, we nally obtain
|x
m
x
n
|
q
n2
1 q
|x
3
x
2
| = cq
n
, where c =
|x
3
x
2
|
q
2
(1 q)
whenever m > n 2.
Since q < 1, q
n
0. Lemma 1.6 ensures now that {x
n
} is a Cauchy sequence. By Theo-
rem 1.4, {x
n
} converges.
16
2 Uniformly continuous functions
Denition 2.1. Let A be a subset of R and f : A R. We say that the function f is
uniformly continuous if for each > 0, there is > 0 such that |f(x) f(y)| < for any
x, y A satisfying |x y| < .
Of course, any uniformly continuous function is continuous.
Theorem 2.2. Let I be an interval and f : I R be a uniformly continuous function.
Then f is continuous at every x
0
I.
Proof. Let x
0
I and > 0. Since f is uniformly continuous, there is > 0 such that
|f(x) f(y)| < whenever x, y I and |x y| < . We apply this property with y = x
0
.
Then |f(x) f(x
0
)| < whenever x I and |x x
0
| < . By denition, f is continuous at
x
0
.
In particular, if you wish to gure out whether a given function f is uniformly continuous
and f happens to have a discontinuity point, you can safely conclude that f is not uniformly
continuous. If f is continuous and we need to show that it is NOT uniformly continuous,
one has to use the NEGATION of uniform continuity. Straight from Denition 2.1, we see
that
Proposition 2.3. For A R and f : A R, f is NOT uniformly continuous, if there exists
> 0 such that for any > 0, there exist x, y A for which |xy| < and |f(x)f(y)| .
Next, we provide few examples of continuous functions, some of which turn out to be
uniformly continuous, while the others are not.
Example 2.4. 1. Let a, b R. Then the linear function f : R R, f(x) = ax + b for each
x R is uniformly continuous.
2. The functions f, g : R R, f(x) = sin x and g(x) = cos x are uniformly continuous.
3. The function f : [0, ) R, f(x) =

x is uniformly continuous.
4. The function f : [1, ) R, f(x) = ln x is uniformly continuous.
5. The function f : (0, ) R, f(x) = ln x is not uniformly continuous.
6. The function f : R R, f(x) = x
2
is not uniformly continuous.
7. The function f : R R, f(x) = e
x
is not uniformly continuous.
8. The function f : (, 0] R, f(x) = e
x
is uniformly continuous.
9. The function f : (0, 1) R, f(x) =
1
x
is not uniformly continuous.
10. The function f : (0, 1) R, f(x) = sin(1/x) is not uniformly continuous.
Proof. 1. Let > 0. Take any > 0 satisfying |a| < . For instance, we can take = 1 if
a = 0 and =

2|a|
otherwise. Then for any x, y R satisfying |x y| < , we have
|f(x) f(y)| = |(ax + b) (ay + b)| = |a||x y| |a| < .
Hence f is uniformly continuous.
2. Let > 0. Take = . Let x, y R be such that |xy| < (= ). Since | sin xsin y| =
2| sin(
xy
2
) cos(
x+y
2
)| and | cos x cos y| = 2| sin(
xy
2
) sin(
x+y
2
)|, we have | sin x sin y|
2| sin(
xy
2
)| and | cos x cos y| 2| sin(
xy
2
) (we use the inequalities | cos(
x+y
2
)| 1 and
17
| sin(
x+y
2
)| 1). Since | sin t| |t| for any t R, we get | sin x sin y| 2| sin(
xy
2
)|
2
|xy|
2
= |x y| and | cos x cos y| 2| sin(
xy
2
) 2
|xy|
2
= |x y|. Hence
|f(x)f(y)| = | sin xsin y| |xy| < = and |g(x)g(y)| = | cos xcos y| |xy| < =
whenever |x y| < . Hence f and g are uniformly continuous.
3. Let > 0. Take =
2
. Let x, y [0, ) be such that |x y| < . Swapping x and y,
if necessary, we can assume that x y. Then y = x + h with 0 h < =
2
. Then
|f(x) f(y)| =

y

x <

x +
2

x =
(

x +
2

x)(

x +
2
+

x)

x +
2
+

x
=
=
(

x +
2
)
2
(

x)
2

x +
2
+

x
=

2

x +
2
+

2
= .
That is, |f(x) f(y)| < whenever, x, y [0, ) and |x y| < . Thus f is uniformly
continuous.
4. Let > 0. Take = . Let x, y [0, ) be such that |x y| < . Swapping x and y,
if necessary, we can assume that x y. Then y = x + h with 0 h < . Then
|f(x) f(y)| = ln y ln x = ln
y
x
< ln
x +
x
= ln
_
1 +

x
_
.
Since x 1, we have

x
and by the above display,
|f(x) f(y)| < ln
_
1 +

x
_
ln(1 + ).
Using the inequality ln(1+t) t for t 0, we get |f(x) f(y)| < ln(1+) = . That is,
|f(x) f(y)| < whenever, x, y [1, ) and |x y| < . Thus f is uniformly continuous.
5. For any > 0, we can choose n N such that e
n
< . Then for x = e
n
and y = e
n1
(which are both in (0, )), we have |x y| = e
n
e
n1
< e
n
< , while f(x) = n,
f(y) = n 1 and therefore |f(x) f(y)| = 1 1. Thus Proposition 2.3 (with = 1)
implies that f is not uniformly continuous.
6. For any > 0, we can choose n N such that
1
n
< . Then for x = n and y = n +
1
n
,
we have |x y| =
1
n
< , while f(x) = n
2
, f(y) = (n +
1
n
)
2
= n
2
+ 2 +
1
n
2
and therefore
|f(x)f(y)| = 2+
1
n
2
2. Thus Proposition 2.3 (with = 2) implies that f is not uniformly
continuous.
7. For any > 0, we can choose n N such that
1
n
< . Then for x = n and y = n+
1
n
, we
have |xy| =
1
n
< , while f(x) = e
n
, f(y) = e
n
e
1/n
and therefore |f(x)f(y)| = e
n
(e
1/n
1).
Since e
1/n
1
1
n
and e
n
n, we get |f(x) f(y)| 1. Thus Proposition 2.3 (with = 1)
implies that f is not uniformly continuous.
8. Let > 0. Take = . Let x, y (, 0] be such that |x y| < . Swapping x and
y, if necessary, we can assume that x y. Then y = x h with 0 h < . Then
|f(x) f(y)| = e
x
e
y
= e
x
e
xh
= e
x
(1 e
h
) < e
x
(1 e

).
Since x 0, e
x
1. Then using the above display and the inequality 1 e
t
t for t 0,
we get |f(x) f(y)| < = whenever, x, y (, 0] and |x y| < . Thus f is uniformly
continuous.
18
9. For any > 0, we can choose n N such that
1
n
< . Then for x =
1
n
and y =
1
n+1
,
we have |x y| <
1
n
< , while f(x) = n, f(y) = n + 1 and therefore |f(x) f(y)| = 1 1.
Thus Proposition 2.3 (with = 1) implies that f is not uniformly continuous.
10. For any > 0, we can choose n N such that
1
n
< . Then for x =
1
2n
and
y =
1
(2n+1/2)
, we have |xy| <
1
n
< , while f(x) = 0, f(y) = 1 and therefore |f(x)f(y)| =
1 1. Thus Proposition 2.3 (with = 1) implies that f is not uniformly continuous.
As usual, there is a translation of the concept of uniform continuity to the language of
convergent sequences.
Theorem 2.5. Let A R and f : A R. Then f is uniformly continuous if and only if
f(x
n
) f(y
n
) 0 for any two sequences {x
n
}
nN
and {y
n
}
nN
in A satisfying x
n
y
n
0.
Proof. First, assume that f is uniformly continuous. Let {x
n
}
nN
and {y
n
}
nN
be two
sequences in A satisfying x
n
y
n
0. We have to prove that f(x
n
) f(y
n
) 0. We use
the denitions of uniform continuity and of a convergent sequence. Let > 0. Since f is
uniformly continuous, there is > 0 such that
|f(x) f(y)| < for any x, y A satisfying |x y| < .
Since x
n
y
n
0, there is n
0
N such that |x
n
y
n
| < for any n n
0
. By the last
display, we have |f(x
n
) f(y
n
)| < for any n n
0
. Hence f(x
n
) f(y
n
) 0.
Next, assume that f(x
n
) f(y
n
) 0 for any two sequences {x
n
}
nN
and {y
n
}
nN
in
A satisfying x
n
y
n
0. We have to show that f is uniformly continuous. Assume the
contrary. Then by Proposition 2.3, there exists > 0 such that for any > 0, there exist
x, y A for which |x y| < and |f(x) f(y)| . Applying this property with =
1
n
,
for every n N, we nd x
n
and y
n
in A such that |x
n
y
n
| <
1
n
and |f(x
n
) f(y
n
)| .
We got ourselves two sequences {x
n
}
nN
and {y
n
}
nN
in A. The inequality |x
n
y
n
| <
1
n
implies that
1
n
x
n
y
n

1
n
and by the sandwich rule x
n
y
n
0 (we know that
1
n
0
and
1
n
0). By our assumption, f(x
n
) f(y
n
) 0, which contradicts the fact that
|f(x
n
) f(y
n
)| for each n N. This contradiction completes the proof.
In the case when the domain of a function is bounded, there is an alternative version of
the above theorem.
Theorem 2.6. Let A R be bounded and f : A R. Then f is uniformly continuous if
and only if {f(x
n
)}
nN
is a Cauchy sequence for any Cauchy sequence {x
n
}
nN
in A.
Proof. First, assume that f is uniformly continuous. Let {x
n
}
nN
be a Cauchy sequence
in A. We have to prove that {f(x
n
)}
nN
is a Cauchy sequence. We use the denitions of
uniform continuity and of a Cauchy sequence. Let > 0. Since f is uniformly continuous,
there is > 0 such that
|f(x) f(y)| < for any x, y A satisfying |x y| < .
Since {x
n
}
nN
is a Cauchy sequence, there is n
0
N such that |x
n
x
m
| < for any
n, m n
0
. By the last display, we have |f(x
n
) f(x
m
)| < for any n, m n
0
. Hence
{f(x
n
)}
nN
is a Cauchy sequence.
Next, assume that {f(x
n
)}
nN
is a Cauchy sequence whenever {x
n
}
nN
is a Cauchy se-
quence in A. We have to show that f is uniformly continuous. Assume the contrary. That is,
19
f is not uniformly continuous. By Proposition 2.3, there exists > 0 such that for any > 0,
there exist x, y A for which |xy| < and |f(x) f(y)| . Applying this property with
=
1
n
, for every n N, we nd x
n
and y
n
in A such that |x
n
y
n
| <
1
n
and |f(x
n
)f(y
n
)| .
We got ourselves two sequences {x
n
}
nN
and {y
n
}
nN
in A. The inequality |x
n
y
n
| <
1
n
implies that
1
n
x
n
y
n

1
n
and by the sandwich rule x
n
y
n
0 (we know that
1
n
0 and
1
n
0). Since A is bounded, the sequence {x
n
} is bounded. By the Bolzano
Weierstrass theorem, there is a subsequence {x
n
k
}
kN
of {x
n
}
nN
such that x
n
k
t R.
Since y
n
x
n
0, we have y
n
k
x
n
k
0 and therefore y
n
k
= x
n
k
+(y
n
k
x
n
k
) t +0 = t.
Since x
n
k
t and y
n
k
t, the sequence {u
j
}
jN
dened as
(u
1
, u
2
, u
3
, u
4
, . . . ) = (x
n
1
, y
n
1
, x
n
2
, y
n
2
, . . . )
(that is, u
2k
= y
n
k
and u
2k1
= x
n
k
for any k N) also converges to t and therefore {u
j
} is a
Cauchy sequence (Theorem 1.4). On the other hand, |f(u
2k
)f(u
2k1
)| = |f(y
n
k
)f(x
n
k
)|
for every k N. It follows that {f(u
j
)} is not a Cauchy sequence. We have arrived to a
contradiction.
Example 2.4 provides plenty of continuous not uniformly continuous functions dened on
an interval of the real line. In all these examples the interval was either unbounded or not
closed. This happened for a reason, as follows from the next theorem, which is the main
result of this chapter.
Theorem 2.7. Let f : [a, b] R be a continuous function dened on the closed bounded
interval [a, b]. Then f is uniformly continuous.
Proof. Let {x
n
}
nN
be a Cauchy sequence in [a, b]. By Theorem 2.6, it suces to show that
{f(x
n
)}
nN
is a Cauchy sequence.
By Theorem 1.4, x
n
t R. Since a x
n
b for every n, we have a t b. That is
x
n
t [a, b]. Since f is continuous at t, f(x
n
) f(t). Since every convergent sequence is
a Cauchy sequence (Theorem 1.4), {f(x
n
)} is a Cauchy sequence. The proof is complete.
20
3 Series
Informally, a series is the problem which arises when we attempt to add together the terms
of a sequence. So, if a = {a
n
}
nN
= (a
1
, a
2
, a
3
, a
4
, . . . ) is a sequence, then the corresponding
series is
a
1
+ a
2
+ a
3
+ a
4
+ . . .
written for short as

i=1
a
i
.
But this is too informal to work with; what meaning (if any!) can be given to this? It is
certainly not ordinary addition, which ONLY makes sense for a nite number of terms.
Suppose that we put
s
1
= a
1
s
2
= a
1
+ a
2
s
3
= a
1
+ a
2
+ a
3
s
4
= a
1
+ a
2
+ a
3
+ a
4
etc.
In other notation
s
n
=
n

j=1
a
j
.
This produces a new sequence {s
n
}
nN
= (s
1
, s
2
, s
3
, s
4
, . . .), the sequence of partial sums. If
this sequence has a limit S, it seems reasonable to regard S as the sum to innity of the
series. So this is how we set up the proper denition:
Denition 3.1. If {a
n
}
nN
= (a
1
, a
2
, a
3
, a
4
, . . .) is a sequence of real (or complex) numbers,
then the series

n=1
a
n
, also written informally as a
1
+a
2
+a
3
+. . ., is said to converge to the
sum s if the sequence
{s
n
}
nN
= (a
1
, a
1
+ a
2
, a
1
+ a
2
+ a
3
, a
1
+ a
2
+ a
3
+ a
4
, . . .) =
_
n

j=1
a
j
_
nN
.
of its partial sums converges to the number s. Otherwise, the series is said to be divergent.
To indicate that the series

n=1
a
n
converges to s, we often write

n=1
a
n
= s and the symbol

n=1
a
n
is often used to denote the sum s, as well as for the series itself.
Sometimes, n = 1 isnt the most convenient starting point. Its OK, for instance, to
abbreviate
2
4
+ 2
5
+ 2
6
+ . . . by

i=4
2
i
or 1 + 2 + 3 + . . . by

n=0
(n + 1).
21
3.1 Geometric Series
For (real or complex) z consider the series

n=1
z
n1
= 1 + z + z
2
+ z
3
+ . . .. The n
th
partial
sum is
s
n
= 1 + z + z
2
+ z
3
+ . . . + z
n1
.
Hence
zs
n
= z + z
2
+ z
3
+ . . . + z
n1
+ z
n
.
Subtracting the two last equalities from each other, we get (1 z)s
n
= 1 z
n
and therefore
s
n
=
1 z
n
1 z
if z = 1.
It is also obvious that s
n
= n if z = 1. Now since z
n
0 if |z| < 1, we have s
n

1
1z
. Since
{z
n
} diverges if |z| 1, see that {s
n
} diverges as well (check it, by considering two cases
z = 1 and z = 1). Thus

n=1
z
n1
is convergent (with sum
1
1z
) if and only if |z| < 1.
3.2 Telescoping or collapsing series
Proposition 3.2. Assume that {b
n
}
nN
is a sequence of (real or complex) numbers such
that b
n
0. For n N let a
n
= b
n
b
n+1
. Then the series

n=1
a
n
converges to b
1
.
Proof. For n N, let s
n
be the n
th
partial sum of the series

n=1
a
n
. Then
s
n
= a
1
+ a
2
+ a
3
+ . . . + a
n1
+ a
n
=
= (b
1
b
2
) + (b
2
b
3
) + (b
3
b
4
) + . . . + (b
n1
b
n
) + (b
n
b
n+1
) =
= b
1
+ (b
2
+ b
2
) + (b
3
+ b
3
) + . . . + (b
n1
+ b
n
) b
n+1
= b
1
b
n+1
.
Since b
n
0, we have s
n
b
1
. That is,

n=1
a
n
converges to b
1
.
Example 3.3. The series

n=1
1
n
2
+n
converges to 1.
Proof. Let a
n
=
1
n
2
+n
. It is easy (partial fractions) to see that a
n
=
1
n

1
n+1
. Hence
a
n
= b
n
b
n+1
with b
n
=
1
n
. Clearly b
n
0 and by Proposition 3.2,

n=1
1
n
2
+n
converges to
b
1
= 1.
We can easily generalize the above proposition.
Proposition 3.4. Assume that {b
n
}
nN
is a sequence of (real or complex) numbers such
that b
n
0 and k N. For n N let a
n
= b
n
b
n+k
. Then the series

n=1
a
n
converges to
b
1
+ . . . + b
k
.
22
Proof. For n N, n > k let s
n
be the n
th
partial sum of the series

n=1
a
n
. Then
s
n
= a
1
+ a
2
+ a
3
+ . . . + a
n1
+ a
n
=
= (b
1
b
k+1
) + (b
2
b
k+2
) + (b
3
b
k+3
) + . . . + (b
n1
b
n+k1
) + (b
n
b
n+k
) =
= b
1
+ . . . + b
k
+ b
k+1
+ . . . + b
n
(b
k+1
+ . . . + b
n
+ b
n+1
+ . . . + b
n+k
)
The sum b
k+1
+ . . . + b
n
cancels out and we are left with
s
n
= b
1
+ . . . + b
k
(b
n+1
+ . . . + b
n+k
).
Since b
n
0, we have s
n
b
1
+ . . . + b
k
. That is,

n=1
a
n
converges to b
1
+ . . . + b
k
.
Example 3.5. The series

n=1
1
n
2
+2n
converges to 3/4.
Proof. Let a
n
=
1
n
2
+2n
. Then (use partial fractions) a
n
=
1
2n

1
2n+4
= b
n
b
n+2
, where
b
n
=
1
2n
. Clearly b
n
0 and therefore by Proposition 3.4, the series

n=1
1
n
2
+2n
converges to
b
1
+ b
2
=
1
2
+
1
4
=
3
4
.
Example 3.6. The series

n=1
1
n
3
+3n
2
+2n
converges to 1/4.
Proof. Let a
n
=
1
n
3
+3n
2
+2n
. Then (use partial fractions) a
n
= b
n
b
n+1
with b
n
=
1
2(n
2
+n)
.
By Proposition 3.2, the series

n=1
1
n
3
+3n
2
+2n
converges to b
1
=
1
4
.
3.3 The Harmonic Series
The harmonic series is

n=1
1
n
= 1 +
1
2
+
1
3
+
1
4
+
1
5
+ . . .
A somewhat reasonable person might expect this to converge. IT DOES NOT.
Look at s
2
n, the sum of the rst 2
n
terms.
s
2
n = 1 +
1
2
+ (
1
3
+
1
4
) + (
1
5
+
1
6
+
1
7
+
1
8
) + . . . +
_
1
2
n1
+ 1
+ . . . +
1
2
n
_
>
> 1 +
1
2
+ (
1
4
+
1
4
) + (
1
8
+
1
8
+
1
8
+
1
8
) + . . . +
_
1
2
n
+ . . . +
1
2
n
_
=
= 1 +
1
2
+
1
2
+
1
2
+ . . . +
1
2
= 1 +
n
2
.
Thus {s
2
n} is unbounded. Hence {s
n
} is unbounded and therefore {s
n
} is not convergent.
23
3.4 Elementary properties of convergent series
We get several results on series for free, just by applying the known properties of convergent
sequences to the sequence {s
n
} of partial sums.
Theorem 3.7. (1) No series can have more than one sum.
(2) If

k=1
a
k
and

k=1
b
k
converge to and m respectively, then

k=1
(a
k
+ b
k
) converges to
+ m and

k=1
(a
k
) converges to for each constant .
Proof. (1) follows directly from the uniqueness of limits for sequences.
(2) follows from the additivity (sum of two convergent sequences converges to the sum of
the limits) of the limit and the relation of the limit with the multiplication by a constant.
Corollary 3.8. If the series

k=1
a
k
converges, then a
k
0.
Proof. Since

k=1
a
k
converges, s
n
s, where s
n
= a
1
+ . . . + a
n
are the partial sums. Then
s
n1
s. Since for n 2, a
n
= s
n
s
n1
, we, using the arithmetic properties of limits,
have
lim
n
a
n
= lim
n
(s
n
s
n1
) = lim
n
s
n
lim
n
s
n1
= s s = 0
as required.
Warning. The converse is FALSE! That is, a
k
0 is NOT enough to guarantee

a
k
convergent: look at the harmonic series again!
Given a series

k=1
a
k
, if we change the rst term from a
1
to a

1
then we will add (a

1
a
1
)
to each partial sum. This will alter the numerical value of the sum to innity (if it exists),
but it wont create or destroy convergence. The same remarks apply if we change any nite
number of terms. So:
Theorem 3.9. If

k=1
a
k
and

k=1
b
k
are series, and there is an n
0
N such that a
k
= b
k
for
every k > n
0
, then either they both converge or they both diverge.
Note that the same applies if we add/remove nitely many terms to/from an innite series.
Thus if we only wish to decide whether or not a series has a sum, we can always aord to
ignore or change any nite number of its terms.
3.5 Series of non-negative reals
If each a
k
is 0, then s
n+1
s
n
= a
n+1
0, where s
m
= a
1
+ . . . + a
m
are the partial sums
of the series

k=1
a
k
. That is, s
n+1
s
n
for any n. Thus {s
n
}

n=1
is an increasing sequence
and its convergence is easier to discuss.
24
Theorem 3.10. If a
k
0 for any k N, then

k=1
a
k
converges if and only if the sequence
{s
k
}

k=1
of its partial sums is bounded (that is, there is a constant K > 0 such that s
n
=
n

1
a
k
K for all n).
Proof. Since {s
n
} is increasing, {s
n
} converges if and only if it is bounded.
Note that a monotonic sequence is bounded if and only if it has a bounded subsequence.
Thus boundedness of {s
k
}

k=1
in the above theorem can be replaced by the requirement for
{s
k
}

k=1
to have a bounded subsequence.
The following theorem is known as the COMPARISON TEST.
Theorem 3.11. Let

k=1
a
k
and

k=1
b
k
be two series of positive real numbers such that

k=1
a
k
converges and the sequence {
b
k
a
k
}

k=1
is bounded. Then the series

k=1
b
k
converges.
Proof. Convergence of

k=1
a
k
implies convergence of the sequence A
k
=
k

j=1
a
j
of the partial
sums. Since every convergent sequence is bounded, there is K > 0 such that A
k
K. Since
{
b
k
a
k
}

k=1
is bounded, there is C > 0 such that
b
k
a
k
C or equivalently b
k
Ca
k
for each
k N. Then the partial sums B
k
of

k=1
b
k
satisfy
B
k
= b
1
+ . . . + b
k
Ca
1
+ . . . + Ca
k
= C(a
1
+ . . . + a
k
) = CA
k
CK.
Hence the sequence {B
k
}
kN
of partial sums of

k=1
b
k
is bounded and therefore

k=1
b
k
is
convergent by Theorem 3.10.
Since every convergent sequence is bounded, we have the following corollary.
Corollary 3.12. Let

k=1
a
k
and

k=1
b
k
be two series of positive real numbers such that

k=1
a
k
converges and the sequence {
b
k
a
k
}

k=1
converges. Then the series

k=1
b
k
converges.
The next result, known as the LIMIT COMPARISON TEST, follows easily.
Theorem 3.13. Let

k=1
a
k
and

k=1
b
k
be two series of positive real numbers such that
= lim
k
b
k
a
k
exists and is non-zero. Then either the two series both converge or they both diverge.
Proof. By Corollary 3.12, convergence of

k=1
a
k
implies convergence of

k=1
b
k
. Since = 0,
we have lim
k
a
k
b
k
exists (and equals 1/). Applying Corollary 3.12 with the roles of

k=1
a
k
and

k=1
b
k
exchanged, we see that convergence of

k=1
b
k
implies convergence of

k=1
a
k
. Thus
either the two series both converge or they both diverge.
25
Example 3.14. The series

k=1
1
k
2
converges.
Proof. Let a
k
=
1
k
2
and b
k
=
1
k
2
+k
. Example 3.3 shows that

k=1
b
k
converges (to 1). On the
other hand,
lim
k
a
k
b
k
= lim
k
k
2
+ k
k
2
= lim
k
_
1 +
1
k
_
= 1.
By the limit comparison test

k=1
a
k
=

k=1
1
k
2
converges.
Example 3.15. The series

n=1
2n
2
1
5+n+n
3
diverges.
Proof. If a
n
=
2n
2
1
5+n+n
3
and b
n
=
1
n
, then
a
n
b
n
2 as n . Also

n=1
b
n
(it is the harmonic
series) diverges. By the limit comparison test

n=1
a
n
diverges.
The comparison and the limit comparison test allow to reduce the question of convergence
of a given series to the question of convergence of another (known or simpler) series.
3.5.1 Condensation test
Here we formalize the method in which we have proved that the harmonic series diverges.
The following criterion turns out to be very handy.
Theorem 3.16. Let {a
n
}

n=1
be a monotonically decreasing sequence of non-negative num-
bers. Then

n=1
a
n
converges if and only if

k=1
2
k
a
2
k converges.
Proof. First, assume that

n=1
a
n
converges. Then the sequence s
n
= a
1
+ . . . + a
n
of its
partial sums converges and therefore is bounded. We can write
s
2
k
1
= a
1
+ (a
2
+ a
3
) + (a
4
+ a
5
+ a
6
+ a
7
) + . . . + (a
2
k1 + . . . + a
2
k
1
).
That is, s
2
k1 = b
0
+ b
1
+ . . . + b
k1
, where b
j
= a
2
j + . . . + a
2
j+1
1
. Since a
n
is decreasing,
each term in the last sum is greater than a
2
j+1 and there are 2
j
of them. Hence b
j
2
j
a
2
j+1.
Equivalently 2
j+1
a
2
j+1 2b
j
. Using these inequalities, we get
2a
2
+ 4a
4
+ . . . + 2
k
a
2
k 2(b
0
+ b
1
+ . . . + b
k1
) = 2s
2
k
1
.
Since the sequence {s
n
} is bounded, so is the sequence 2a
2
+4a
4
+. . . +2
k
a
2
k of partial sums
of the series

2
j
a
2
j . Since the latter is increasing, it converges. Thus

k=1
2
k
a
2
k converges.
Assume now that

k=1
2
k
a
2
k converges. Using once again the fact that a
n
is decreasing, we
see that each term in the sum dening b
j
does not exceed a
2
j . Hence b
j
2
j
a
2
j . It follows
that
s
2
k1 = b
0
+ b
1
+ . . . + b
k1

k1

j=0
2
j
a
2
j .
26
The right-hand side in the above display is a partial sum of the convergent series

j=1
2
j
a
2
j
and therefore is bounded. Hence the sequence {s
2
k1}
kN
is bounded. Thus the increasing
sequence {s
n
}
nN
has a bounded subsequence and therefore is bounded itself. Hence {s
n
}
nN
converges (as a bounded increasing sequence) and so does the series

n=1
a
n
.
Theorem 3.17. Let p be a real constant. Then

k=1
k
p
if and only if p < 1.
Proof. Let a
k
= k
p
. If p 0, then a
k
1 and therefore a
k
0 and the series

k=1
a
k
diverges.
If p < 0, then {a
k
} is decreasing. By Theorem 3.16,

k=1
a
k
converges if and only if

n=1
b
n
converges, where b
n
= 2
n
a
2
n = 2
n
(2
n
)
p
= (2
1p
)
n
. The series

n=1
b
n
is a geometric series with
the common ratio 2
1p
. Thus it converges if and only if 2
1p
< 1 p < 1.
Applying Theorem 3.17 with p = 1/2 and p = 3/2, we see, for instance that the series

k=1
1

k
diverges, while the series

k=1
1
k

k
converges.
Example 3.18. The series

n=1
n
2
n+2

n+1(2n
3
n
2
)
converges.
Proof. Let a
n
n
2
n+2

n+1(2n
3
n
2
)
and b
n
=
1
n
3/2
. Direct computation shows that
b
n
a
n
2. By
Theorem 3.17,

n=1
b
n
converges. It remains to notice that a
n
, b
n
are positive and apply the
limit comparison test to conclude that

n=1
a
n
=

n=1
n
2
n+2

n+1(2n
3
n
2
)
converges.
Next, we prove the so n
th
root test and the ratio test. These tests are applications of the
comparison test with one of the series being a geometric series.
3.5.2 The n
th
root test
Theorem 3.19. Let

k=1
a
k
be a series of positive reals such that the sequence {
n

a
n
} con-
verges to some limit .
(i) If < 1, then

a
k
converges.
(ii) If > 1, then a
k
0 and therefore

a
k
diverges.
Proof. Let < 1 and put =
1
2
> 0. Since
n

a
n
, there is n
0
N such that n n
0
guarantees <
n

a
n
< + . Hence
n

a
n
< 1 (see denition of ) and therefore
a
n
< (1 )
n
for n n
0
. Let b
n
= (1 )
n
. Since
a
n
b
n
1 for n n
0
, the sequence {
a
n
b
n
}
nN
is bounded. Since 1 < 1, the series

n=1
b
n
=

n=1
(1 )
n
converges. By the comparison
test (Theorem 3.11),

k=1
a
k
also converges.
27
Let > 1 and put =
1
2
> 0. Since
n

a
n
, there is n
0
N such that n n
0
guarantees <
n

a
n
< + . Hence 1 + <
n

a
n
and therefore a
n
> (1 + )
n
> 1 for
n n
0
. Clearly a
n
0 and therefore

n=1
a
n
diverges.
Warning. If
n

a
n
= 1 the n
th
root test is inconclusive. Try another test.
Example 3.20. The series

n=1
_
1
1
n+1
_
n
2
converges.
Proof.
n
_
_
1
1
n + 1
_
n
2
=
_
n
n + 1
_
n
=
1
(1 +
1
n
)
n

1
e
< 1.
Example 3.21. Let t > 0. Then the series

n=1
n
n+2
(2n+1)
n
t
n
converges if and only if t < 2.
Proof. Let a
n
=
n
n+2
(2n+1)
n
t
n
. Then
lim
n
(a
n
)
1/n
= lim
n
n n
2/n
2n + 1
t = t/2.
By the n
th
root test the series converges if t/2 < 1 (that is, t < 2) and diverges if t/2 > 1
(that is, t > 2). It remains to consider the case t = 2.
Show yourself that in this case a
n
0 (and therefore

n=1
a
n
diverges.)
3.5.3 The Ratio Test = dAlemberts Test
Theorem 3.22. Let

k=1
a
k
be a series of positive terms such that {
a
k+1
a
k
} converges to a limit
.
(i) If < 1, then

k=1
a
k
converges.
(ii) If > 1, then a
k
0 and therefore

k=1
a
k
diverges.
Proof. If < 1, we put =
1
2
> 0. Since
a
n+1
a
n
, we can pick n
0
N such that
a
n+1
a
n
< 1 for n n
0
. Then for each m > n
0
, we write
a
m
a
n
0
=
a
m
a
m1

a
m1
a
m2
. . .
a
n
0
+1
a
n
0
.
Each ratio in the right-hand side is less than 1 and there are m n
0
of them. Hence
a
m
a
n
0
< (1 )
mn
0
. Hence
a
m
(1)
m
C =
a
n
0
(1)
n
0
for m n
0
and therefore the sequence {
a
n
b
n
}
is bounded, where b
n
= (1 )
n
. Since 1 < 1, the geometric series

n=1
b
n
converges and
by the comparison test (Theorem 3.11), the series

k=1
a
k
also converges.
28
If > 1, we put =
1
2
> 0. Since
a
n+1
a
n
, we can pick n
0
N such that
a
n+1
a
n
> 1+ > 1
for n n
0
. Then for each m > n
0
, we write
a
m
a
n
0
=
a
m
a
m1

a
m1
a
m2
. . .
a
n
0
+1
a
n
0
.
Each ratio in the right-hand side is greater than 1 and therefore so is their product. Hence
a
m
a
n
0
1 for m > n
0
. That is, a
m
> a
n
0
for m > n
0
. Since a
n
0
> 0, we have a
m
0. Thus

m=1
a
m
diverges.
Warning. Again, if
a
n+1
a
n
1, the ratio test inconclusive: try something else.
Example 3.23. The series

n=1
n!
n
n
converges.
Proof. Let a
n
=
n!
n
n
. Then
a
n+1
a
n
=
(n + 1)!n
n
n!(n + 1)
n+1
=
1
(1 + n
1
)
n
e
1
.
Since e
1
< 1, the ratio test ensures convergence of

n=1
a
n
.
Example 3.24. Let t > 0. Then the series

n=1
(n!)
2
t
n
(2n)!
converges if and only if t < 4.
Proof. Let a
n
=
(n!)
2
t
n
(2n)!
. Then
a
n+1
a
n
=
((n + 1)!)
2
t
n+1
(2n)!
(2n + 2)!(n!)
2
t
2
=
(n + 1)
2
t
(2n + 2)(2n + 1)

t
4
.
By the ratio test, if 0 < t < 4,

n=1
a
n
is convergent; if t > 4,

n=1
a
n
is divergent.
If t = 4, then
a
n+1
a
n
=
4n
2
+ 8n + 4
4n
2
+ 4n + 2
> 1
so a
n+1
> a
n
and the sequence {a
n
} is increasing and cannot tend to zero. Thus

n=1
a
n
is
divergent again.
3.6 Absolute convergence
How do we cope with

k=1
a
k
when some a
k
s are negative or even complex? Take modulus!
Denition 3.25. A series

k=1
a
k
is absolutely convergent when

k=1
|a
k
| is convergent. A
convergent series which is not absolutely convergent is called conditionally convergent.
29
This is a proper place to introduce the Cauchy criterion of convergence.
Theorem 3.26. The series

k=1
a
k
converges if and only if for every > 0, there is n
0
N
such that

n+p

j=n
a
j

= |a
n
+ a
n+1
+ . . . + a
n+p
| <
whenever p, n are integers, p 0 and n > n
0
.
Proof. Let s
n
= a
1
+ . . . + a
n
be the partial sums of the series

k=1
a
k
. By denition the last
series converges if and only if the sequence {s
n
} converges. By Theorem 1.4, {s
n
} converges
if and only if {s
n
} is a Cauchy sequence. By denition of a Cauchy sequence, {s
n
} is a
Cauchy sequence if and only if for every > 0 there is n
0
N such that |s
k
s
l
| < for all
k, l n
0
. If n = 1 + min{k, l} and p = |k l| 1, then
|s
k
s
l
| = |(a
1
+ . . . + a
k
) (a
1
+ . . . + a
l
)| = |a
n
+ . . . + a
n+p
|
(since n + p = max{k, l} and all terms a
j
with j < n in the above dierence cancel out).
Thus {s
k
}
kN
is a Cauchy sequence if and only if for each > 0, there is n
0
N such that

n+p

j=n
a
j

= |a
n
+ a
n+1
+ . . . + a
n+p
| <
whenever p, n are integers, p 0 and n > n
0
.
Theorem 3.27. If

k=1
a
k
is absolutely convergent, then it is convergent.
Proof. Applying the Cauchy criterion (Theorem 3.26) to the convergent series

k=1
|a
k
|, we
see that given > 0, there is n
0
N such that |a
n
| + |a
n+1
| + . . . + |a
n+p
| < if p 0,
n > n
0
. The latter (via the triangle inequality) implies that |a
n
+ a
n+1
+ . . . + a
n+p
| < if
p 0, n > n
0
. Now the Cauchy criterion says that

k=1
a
k
is also convergent.
Example 3.28. Let z R (or C). Then the series

n=1
(n!)
2
z
n
(2n)!
converges if and only if |z| < 4.
Proof. Let a
n
=
(n!)
2
z
n
(2n)!
. Then |a
n
| =
(n!)
2
|z|
n
(2n)!
. By Example 3.24 (put t = |z|),

n=1
|a
n
|
converges if and only if |z| < 4. Moreover, |a
n
| 0 and therefore a
n
0 if |z| 4. Hence

k=1
a
n
diverges if |z| 4. If |z| < 4 we use Theorem 3.27 to conclude that

n=1
a
n
converges
because

n=1
|a
n
| does.
30
3.7 Power Series
The series of the form

n=0
a
n
z
n
= a
0
+ a
1
z + a
2
z
2
+ a
3
z
3
+ . . .
is called a Power Series and the terms of the sequence {a
n
}
n0
are called its coecients.
Example 3.28 illustrates what usually happens in these: there is a circle in the complex plane
inside which

converges, and outside which it diverges. Its radius is called the Radius of
Convergence (it is 4 in the above example). In order to replace usual by always one has
to allow zero and innite convergence radii. Zero convergence radius corresponds to the case
when the above series converges only for z = 0 and innite convergence radius corresponds
to convergence for any complex z.
The next theorem provides means of computing the convergence radius in certain circum-
stances.
Theorem 3.29. For a power series

a
n
z
n
, if = lim
n
_
|a
n
| exists then:
(i) if = 0,

a
n
z
n
converges for any z R (or C);
(ii) if > 0,

a
n
z
n
converges for |z| < 1/ and diverges for |z| > 1/.
Proof. Let b
n
= a
n
z
n
. Then |b
n
|
1/n
|z|. By the n
th
root test,

|b
n
| converges if |z| < 1
and |b
n
| 0 if |z| > 1. Using Theorem 3.27, we see that

b
n
converges if |z| < 1 and
diverges if |z| > 1. The result follows.
Remark. There is still a result of this type even if the sequence
n
_
|a
n
| diverges. One
has just to replace lim
n
_
|a
n
| with the number lim
n
sup
kn
k
_
|a
k
|, which is always dened and
coincides with lim
n
_
|a
n
|, when the latter exists.
Example 3.30. The series
exp(z) = 1 + z +
z
2
2!
+
z
3
3!
+ = 1 +

n=1
z
n
n!
converges for any real (or complex) z.
Proof. Let a
n
=
z
n
n!
. If z = 0, the series obviously converges. If z = 0, then
|a
n+1
|
|a
n
|
=
|z|
n + 1
0
and

|a
n
| converges by the ratio test. Hence

a
n
converges absolutely and therefore
converges.
Example 3.31. The series

n=0
(2)
n
(n
2
+ 1)
n + 1
z
n
converges if and only if |z| < 1/2.
31
Proof. Let a
n
=
(2)
n
(n
2
+1)
n+1
z
n
. If |z| 1/2, |a
n
|
n
2
+1
n+1
1 and the series

a
n
diverges
since a
n
0. If |z| < 1/2, then
|a
n+1
|
|a
n
|
=
2|z|((n + 1)
2
+ 1)(n + 1)
(n + 2)(n
2
+ 1)
2|z| < 1
and

|a
n
| converges by the ratio test.
3.8 Alternating Series Test
Theorem 3.32. If {a
k
} is a decreasing sequence of non-negative numbers and a
k
0, then
a
1
a
2
+ a
3
a
4
+ . . . =

k=1
(1)
k+1
a
k
converges.
Proof. The even partial sums
s
2n
= a
1
+ (a
2
+ a
3
) + (a
4
+ a
5
) + . . . + ( ) a
2n
are all a
1
, and they form an increasing sequence because s
2n+2
s
2n
= a
2n+1
a
2n+2
0.
So they converge to some limit . Likewise, the odd partial sums are decreasing and converge
to a limit

. Now s
2n+1
s
2n
= a
2n+1
, so

= lima
2n+1
= 0. That is, =

. Hence

a
k
converges to .
Example 3.33. By Theorem 3.32, the series

n=1
(1)
n
n
= 1
1
2
+
1
3

1
4
+. . . converges, while
the series

n=1

(1)
n
n

n=1
1
n
is the harmonic series and therefore diverges.
Remark. The sum of the above series turns out to be ln 2 = log
e
2:

n=1
(1)
n
n
= ln 2, but
this is not obvious.
3.9 Bracketing, rearrangements and products
Given a series a
1
+a
2
+a
3
+. . . we can create lots of new ones by grouping its terms together,
e.g.
(a
1
+ a
2
) + a
3
+ (a
4
+ a
5
+ a
6
) + (a
7
+ a
8
) + . . .
Theorem 3.34. If

a
k
converges to , then so does any bracketed series obtained from it
(and to the same limit).
Proof. The sequence of partial sums of a bracketed series is a subsequence of partial sums of
the original series. It remains to use the fact that any subsequence of a convergent sequence
converges to the same limit.
Example 3.35. Since 1+
1
2
+
1
4
+
1
8
+. . . = 2, the same holds for 1+(
1
2
+
1
4
)+
1
8
+(
1
16
+
1
32
)+. . .
That is,
1 +
3
4
+
1
8
+
3
32
+
1
64
+
3
256
+ . . . = 2.
32
Example 3.36. Beware that the converse is false! For instance:
(1 1) + (1 1) + (1 1) + . . . = 0 + 0 + 0 + . . . = 0,
yet the unbracketed series 1 1 + 1 1 + 1 1 + . . . does not converge.
Remark. The converse of Theorem 3.34 is OK under any of the following additional
assumptions (DIY). a) a
n
0 for any n; b) a
n
0 and the number of terms in the brackets
is bounded.
When a series converges, you would expect to be able to add up its terms in a dierent
order and get the same total. You would sometimes be wrong! For example, start with
the series

n=1
(1)
n
n
. We know it converges by the alternating series test. Moreover after an
obvious bracketing, we see that
=
_
1
1
2
_
+
_
1
3

1
4
_
+ . . .
is the sum of a series with positive terms and therefore > 0. Now if after rearranging the
terms of the series thus
1
1
2

1
4
+
1
3

1
6

1
8
+
1
5

1
10

1
12
+
1
7
. . .
we still get convergence to , then to also converges
(1
1
2
)
1
4
+ (
1
3

1
6
)
1
8
+ (
1
5

1
10
)
1
12
+ . . .
The latter series can be also written as
1
2

1
4
+
1
6

1
8
+
1
10

1
12
+ . . .
whose terms a halves of the terms of the original series and which therefore converges to
/2 = .
Conclusion Rearranging a convergent series can produce one whose sum is dierent!
Fortunately, there is an important range of instances when it denitely cant.
Theorem 3.37. Any rearrangement of an absolutely convergent series is still convergent
absolutely and to the same sum.
Proof. Technical and not examinable.
Remark. If

a
n
is a conditionally convergent series of real numbers, then ANY real
number can be obtained as its sum after a suitable rearrangement of its terms. This fact is
known as the Riemann Theorem.
Next, we address the question whether we can multiply series. If

n=0
a
n
and

n=0
b
n
are two
series, then their Cauchy product is the series

n=0
c
n
, where c
n
= a
0
b
n
+a
1
b
n1
+. . . +a
n
b
0
=
n

j=0
a
j
b
nj
. In other words c
n
is obtained by formal multiplication of (a
0
+ a
1
+ . . . ) by
(b
0
+b
1
+. . . ), opening the brackets and grouping together a
j
b
k
with the same value of k +j.
33
Theorem 3.38. Let

n=0
a
n
and

n=0
b
n
be two absolutely convergent series. Then their Cauchy
product converges absolutely to the product of the sums of

n=0
a
n
and

n=0
b
n
.
Proof. Technical and not examinable.
The Cauchy product behaves particularly well when we multiply power series. Observe
that if

n=0
a
n
z
n
and

n=0
b
n
z
n
are two power series, then their Cauchy product is again a power
series

n=0
c
n
z
n
, with c
n
= a
0
b
n
+ a
1
b
n1
+ . . . + a
n
b
0
=
n

j=0
a
j
b
nj
. Applying Theorem 3.38,
we obtain the following fact.
Proposition 3.39. Let

n=0
a
n
z
n
and

n=0
b
n
z
n
be two power series with convergence radii r
a
and r
b
respectively and

n=0
c
n
z
n
be their Cauchy product. Then the convergence radius r
c
of

n=0
c
n
z
n
satises r
c
min{r
a
, r
b
} and

n=0
c
n
z
n
=
_

n=0
a
n
z
n
__

n=0
b
n
z
n
_
for any z R (or z C) satisfying |z| < min{r
a
, r
b
}.
Proof. Just take z satisfying |z| < min{r
a
, r
b
} and apply Theorem 3.38 to multiply the
absolutely convergent series

n=0
a
n
z
n
and

n=0
b
n
z
n
.
Example 3.40. Let z, w R (or C). Then exp(z + w) = exp(z) exp(w).
Proof. We apply Theorem 3.38 to the two absolutely convergent series exp(z) =

n=0
z
n
n!
exp(w) =

n=0
w
n
n!
(here for shortness we assume that 0! = 1). We obtain
exp(z) exp(w) =

n=0
c
n
, where c
n
=
n

j=0
z
j
w
nj
j!(n j)!
.
Then
c
n
=
1
n!
n

j=0
n!
j!(n j)!
z
j
w
nj
=
1
n!
n

j=0
_
n
j
_
z
j
w
nj
=
(z + w)
n
n!
,
where we have used the binomial formula. Thus
exp(z) exp(w) =

n=0
c
n
=

n=0
(z + w)
n
n!
= exp(z + w).
The number exp(1) =

n=0
1
n!
is known as the number e. The above example shows, for
instance, that exp(n) = e
n
for any natural number n (and then easily for any rational number
as well) and justies the common notation exp(z) = e
z
.
34
4 Dierentiable Functions
Denition 4.1. Let I be an open interval, x
0
I and f : I R. We say that f is
dierentiable at x
0
if there exists the limit
lim
xx
0
f(x) f(x
0
)
x x
0
= lim
h0
f(x
0
+ h) f(x
0
)
h
.
This limit (when it exists) is written f

(x
0
) (or, in some older textbooks,
df
dx
(x
0
)) and is
called the derivative of f at x
0
.
When f is dierentiable at every point of I, we call it dierentiable on I. In this case f

is a function from I to R.
Theorem 4.2. Let I be an open interval, x
0
I and f : I R be dierentiable at x
0
.
Then f is continuous at x
0
.
Proof. Indeed, for x = x
0
, we have
f(x) = (x x
0
)
_
f(x) f(x
0
)
x x
0
_
+ f(x
0
)
and therefore
lim
xx
0
f(x) = f(x
0
) + lim
xx
0
(x x
0
) lim
xx
0
f(x) f(x
0
)
x x
0
= f(x
0
) + 0 f

(x
0
) = f(x
0
).
That is, f is continuous at x
0
.
Example 4.3. Not all continuous functions are dierentiable. For instance, f(x) = |x| is
easily seen to be continuous on R and is not dierentiable at 0.
Proof. To show that f is not dierentiable at 0 it suces to notice that lim
x0+
f(x)f(0)
x0
= 1,
while lim
x0
f(x)f(0)
x0
= 1. Thus lim
x0
f(x)f(0)
x0
= 1.
There are many properties of dierentiable functions that can be used to compute the
derivatives.
Theorem 4.4. Let f, g : I R with I being an open interval and x
0
I. Assume also that
f and g are dierentiable at x
0
. Then f + g, fg and cf are dierentiable at x
0
for every
c R and
(cf)

(x
0
) = cf

(x
0
), (f +g)

(x
0
) = f

(x
0
)+g

(x
0
) and (fg)

(x
0
) = f

(x
0
)g(x
0
)+f(x
0
)g

(x
0
).
If additionally g(x
0
) = 0, then
f
g
is dierentiable at x
0
and
(f/g)

(x
0
) =
f

(x
0
)g(x
0
) f(x
0
)g

(x
0
)
(g(x
0
))
2
.
35
Proof. We use the arithmetic properties of limits. Clearly
lim
xx
0
(cf)(x)(cf)(x
0
)
xx
0
= lim
xx
0
c
f(x)f(x
0
)
xx
0
= c lim
xx
0
f(x)f(x
0
)
xx
0
= cf

(x
0
),
which proves dierentiability of cf at x
0
and the equality (cf)

(x
0
) = cf

(x
0
).
Next,
lim
xx
0
(f+g)(x)(f+g)(x
0
)
xx
0
= lim
xx
0
_
f(x)f(x
0
)
xx
0
+
g(x)g(x
0
)
xx
0
_
= lim
xx
0
f(x)f(x
0
)
xx
0
+ lim
xx
0
g(x)g(x
0
)
xx
0
= f

(x
0
)+g

(x
0
),
which proves dierentiability of f + g at x
0
and the equality (f + g)

(x
0
) = f

(x
0
) + g

(x
0
).
Next,
lim
xx
0
(fg)(x)(fg)(x
0
)
xx
0
= lim
xx
0
f(x)g(x)f(x
0
)g(x)+f(x
0
)g(x)f(x
0
)g(x
0
)
xx
0
= lim
xx
0
_
g(x)
f(x)f(x
0
)
xx
0
+f(x
0
)
g(x)g(x
0
)
xx
0
_
=
= lim
xx
0
g(x) lim
xx
0
f(x)f(x
0
)
xx
0
+ f(x
0
) lim
xx
0
g(x)g(x
0
)
xx
0
= f

(x
0
)g(x
0
) + f(x
0
)g

(x
0
),
where we used continuity of g at x
0
(Theorem 4.2) to ensure that lim
xx
0
g(x) = g(x
0
). Thus
fg is dierentiable at x
0
and (fg)

(x
0
) = f

(x
0
)g(x
0
) + f(x
0
)g

(x
0
).
Finally, assume that g(x
0
) = 0. Then
lim
xx
0
(f/g)(x)(f/g)(x
0
)
xx
0
= lim
xx
0
f(x)g(x
0
)f(x
0
)g(x)
g(x)g(x
0
)(xx
0
)
= lim
xx
0
f(x)g(x
0
)f(x
0
)g(x
0
)+f(x
0
)g(x
0
)f(x
0
)g(x)
g(x)g(x
0
)(xx
0
)
=
= lim
xx
0
_
1
g(x)
f(x)f(x
0
)
xx
0

f(x
0
)
g(x)g(x
0
)
g(x)g(x
0
)
xx
0
_
= lim
xx
0
1
g(x)
lim
xx
0
f(x)f(x
0
)
xx
0

lim
xx
0
f(x
0
)
g(x)g(x
0
)
lim
xx
0
g(x)g(x
0
)
xx
0
=
f

(x
0
)
g(x
0
)

f(x
0
)g

(x
0
)
(g(x
0
))
2
=
f

(x
0
)g(x
0
)f(x
0
)g

(x
0
)
(g(x
0
))
2
.
Thus f/g is dierentiable at x
0
and (f/g)

(x
0
) =
f

(x
0
)g(x
0
)f(x
0
)g

(x
0
)
(g(x
0
))
2
.
Note that the formula (fg)

= f

g + fg

is known as the Leibniz rule. The next theorem


provides a formula for the derivative of the composition of two functions. It is known as the
chain rule.
Theorem 4.5. Let I and J be open intervals, f : I J is dierentiable at x
0
I and
g : J R is dierentiable at f(x
0
) J. Then the composition g f is dierentiable at x
0
and (g f)

(x
0
) = g

(f(x
0
))f

(x
0
).
Proof. Consider
lim
xx
0
(gf)(x)(gf)(x
0
)
xx
0
= lim
xx
0
(gf)(x)(gf)(x
0
)
f(x)f(x
0
)
f(x)f(x
0
)
xx
0
= lim
xx
0
g(f(x))g(f(x
0
))
f(x)f(x
0
)
lim
xx
0
f(x)f(x
0
)
xx
0
.
The second factor in the last product is just f

(x
0
). Using the change of variable y = f(x)
in the limit, we get lim
xx
0
g(f(x))g(f(x
0
))
f(x)f(x
0
)
= lim
yf(x
0
)
g(y)g(f(x
0
))
yf(x
0
)
= g

(f(x
0
)) by the denition of
the derivative. Thus
lim
xx
0
(g f)(x) (g f)(x
0
)
x x
0
= g

(f(x
0
))f

(x
0
)
and therefore g f is dierentiable at x
0
and (g f)

(x
0
) = g

(f(x
0
))f

(x
0
).
36
Few derivatives are computed by just nding the limit from the denition: (x
a
)

= ax
a1
,
(e
x
)

= e
x
, (sin x)

= cos x, (cos x)

= sin x. The derivatives of the inverse functions of the


exponential and the trigonometric functions follow from the chain rule: (arctan x)

=
1
1+x
2
,
(arcsin x)

=
1

1x
2
, (arccos x)

=
1

1x
2
, (ln x)

=
1
x
. Using Theorems 4.4 and 4.5, it is
possible now to nd explicitly the derivative of any given elementary function. For example,
if f(x) = sin(ln(1 + e
x
2
)), then f

(x) =
cos(ln(1+e
x
2
)) e
x
2
2x
1+e
x
2
.
Remark 4.6. When the derivative f

(x) of f(x) exists at every point of an interval, we may


try dierentiating it. The derivative, if it exists, is written
f

(x)
and called the second derivative of f. In turn, if f

(x) can be dierentiated, its derivative


f

(x) is called the third derivative of f, and so on.


Denition 4.7. If I is an interval, x
0
I and f : I R, then x
0
is called a point of
local maximum for f if there is > 0 such that f(x) f(x
0
) for every x I satisfying
|x x
0
| < .
Similarly, x
0
is called a point of local minimum for f if there is > 0 such that f(x) f(x
0
)
for every x I satisfying |x x
0
| < .
Remark 4.8. It is easy to see that if f attains its maximal value at x
0
I, then x
0
is a
point of local maximum. The same holds true with max replaced by min.
Theorem 4.9. If f : I R has a local maximum or a local minimum at x
0
I and is
dierentiable there, I being an open interval, then f

(x
0
) = 0.
Proof. Case 1: f has a local maximum at x
0
. By denition, there is > 0 such that
f(x) f(x
0
) whenever x I and |x x
0
| < . Since I is open, making smaller if
necessary, we can also assume that (x
0
, x
0
+ ) I. Then for x (x
0
, x
0
+ ), we
have x x
0
> 0 and f(x) f(x
0
) 0. hence
f(x)f(x
0
)
xx
0
0 for x (x
0
, x
0
+ ). Hence
lim
xx
0
+
f(x)f(x
0
)
xx
0
0. On the other hand, for x (x
0
, x
0
), we have x x
0
< 0 and
f(x) f(x
0
) 0. hence
f(x)f(x
0
)
xx
0
0 for x (x
0
, x
0
). Hence lim
xx
0

f(x)f(x
0
)
xx
0
0. Since
one-sided limits coincide with the two sided one when the latter exists:
f

(x
0
) = lim
xx
0
f(x) f(x
0
)
x x
0
= lim
xx
0
+
f(x) f(x
0
)
x x
0
= lim
xx
0

f(x) f(x
0
)
x x
0
.
Thus f

(x
0
) 0 and f

(x
0
) 0. Hence f

(x
0
) = 0.
Case 2: f has a local minimum at x
0
. Then the function f has a local maximum at
x
0
(check it). By Case 1, (f)

(x
0
) = 0. Hence f

(x
0
) = (f)

(x
0
) = 0 = 0.
The above theorem provides means to nd the maximal and minimal values of a continuous
function f : [a, b] R provided f is dierentiable on (a, b).
Theorem 4.10. Let f : [a, b] R be continuous on [a, b] and dierentiable on (a, b). Then
the maximal and minimal values of f on [a, b] are attained on the set S = {a, b} {x
(a, b) : f

(x) = 0}.
37
Proof. Let u and v points in [a, b] at which f attains it maximal and minimal values on [a, b]
respectively. That is, f(v) f(x) f(u) for every x [a, b]. If v / {a, b}, then v is a
point of local maximum for f on the open interval (a, b). Then by Theorem 4.9, f

(v) = 0
and therefore v S. Of course, if v {a, b}, then v S as well. Thus v S in any case.
Similarly u S.
Example 4.11. The function f : [0, 2] R, f(x) = xe
x
attains its maximal value
1
e
at
the point x = 1.
Proof. Since f

(x) = e
x
(1 x), the derivative f

(x) vanishes only when x = 1. By Theo-


rem 4.10 f can only attain its maximal value at a point of the set {0, 1, 2}. Since f(1) =
1
e
and f(0) = 0 <
1
e
, f(2) =
2
e
2
<
1
e
, f attains its maximal value
1
e
at the point x = 1.
4.1 The mean value theorems
The next theorem is known as the Rolle theorem.
Theorem 4.12. Let f : [a, b] R be a continuous function dierentiable on (a, b). Assume
also that f(a) = f(b). Then there exists c (a, b) such that f

(c) = 0.
Proof. Case 1: If f is constant on [a, b] then there is nothing to prove because f

is zero
everywhere.
Case 2: If f takes somewhere a value bigger than f(a), then the place where it reaches its
maximum is neither a nor b but some point c (a, b). Since c is certainly a local maximum,
Theorem 4.9 says f

(c) = 0.
Case 3: Similarly if f takes somewhere a value less than f(a), then the place where it
reaches its minimum is neither a nor b but some point c (a, b). Since c is certainly a local
minimum, Theorem 4.9 says f

(c) = 0.
Example 4.13. The equation
(1 + x
2
) cos x = 2x sin x
has a solution somewhere between 0 and .
Proof. Notice that (1 + x
2
) cos x 2x sin x is the derivative of (1 + x
2
) sin x: an expression
which is continuous and dierentiable on [0, ] and equals zero at 0 and at . Rolle nishes
the proof.
The next theorem is the Cauchy mean value theorem.
Theorem 4.14. Let f, g : [a, b] R be continuous functions, dierentiable on (a, b). Then
there is some c (a, b) for which
(f(b) f(a))g

(c) = (g(b) g(a))f

(c). (4.1)
Proof. Case 1: g(b) = g(a). In this case g satises the conditions of the Rolle Theorem.
Hence there is c (a, b) such that g

(c) = 0. Since g(b) g(a) = 0, (4.1) is trivially satised


with this c.
Case 2: g(b) = g(a). We seek for R to make f +g satisfy the conditions of the Rolle
theorem. That is we need to nd R such that f(a) +g(a) = f(b) +g(b). Solving this
equality and using the relation g(b) = g(a), we obtain =
f(a)f(b)
g(b)g(a)
. Since Rolle applies to
f +g, there exists c (a, b) such that (f +g)

(c) = 0. Hence f

(c) = g

(c). Substituting
=
f(a)f(b)
g(b)g(a)
and multiplying by (g(b) g(a)), we get (4.1).
38
The next theorem is known as the Lagrange mean value theorem.
Theorem 4.15. Let f : [a, b] R be a continuous function, dierentiable on (a, b). Then
there is some c (a, b) for which
f

(c) =
f(b) f(a)
b a
. (4.2)
Proof. Apply Theorem 4.14 with g(x) = x.
4.2 Monotonic functions and the derivative
Theorem 4.16. Let f : [a, b] R be a continuous function, dierentiable on (a, b). Then
f is increasing if and only if f

0 on (a, b). Moreover, f is decreasing if and only if f

0
on (a, b).
Proof. First, assume that f is increasing. Then for any x, y (a, b), x = y,
f(x)f(y)
xy
0.
Passing to the limit as y x we get f

(x) 0. Hence any increasing function has non-


negative derivative.
Let now f

0. Assume that f is not increasing. Then we can nd x, y (a, b) such


that x < y and f(x) > f(y). By the Lagrange Theorem, there exists c (x, y) such that
f

(c) =
f(x)f(y)
xy
< 0. We have arrived to a contradiction.
The equivalence of the decreasing property to f

0 follows from the already proven part


of the theorem and the observation that f is decreasing if and only if f is increasing and
f

0 if and only if (f)

0.
Proposition 4.17. If f is continuous on [a, b] and has zero derivative everywhere in (a, b),
then f is constant on [a, b].
Proof. Assume the contrary. Then there are x, y (a, b) such that x < y and f(x) = f(y).
By the Lagrange Theorem, there exists c (x, y) such that f

(c) =
f(x)f(y)
xy
= 0. We have
arrived to a contradiction.
4.3 The LHopital Rule
The following theorem is one of the versions of the LHopitals Rule.
Theorem 4.18. Let a < c < b and f, g : (a, b) R are dierentiable on (a, b) \ {c}.
Assume also that g

(x) = 0 on (a, b) \ {c}, lim


xc
f(x) = lim
xc
g(x) = 0 and there exists the limit
lim
xc
f

(x)
g

(x)
= R. Then the limit lim
xc
f(x)
g(x)
does exist and equals .
Proof. By setting f(c) = g(c) = 0, if necessary, we can ensure that f and g are continuous
on (a, b). Let u (a, b), u = c. Since f, g are continuous on the closed interval with the ends
u and c and dierentiable inside it, Theorem 4.14 tells us that there is y
u
between c and u
such that (f(u) f(c))g

(y
u
) = (g(u) g(c))f

(y
u
). Since f(c) = g(c) = 0, we get
f(u)
g(u)
=
f

(y
u
)
g

(y
u
)
.
Since y
u
is between c and u, the squeeze principle guarantees that y
u
c as u c. Since
lim
xc
f

(x)
g

(x)
= , we can pass to the limit as u c in the last display to see that lim
xc
f(x)
g(x)
= .
39
Example 4.19. lim
x4
x

x2
4x
=
3
4
.
Proof. Setting f(x) = x

x 2 and g(x) = 4 x, we see that lim


x4
f(x) = lim
x4
g(x) = 0.
Moreover f

(x) = 1
1
2

x
and g

(x) = 1. Then g

does not vanish and


lim
x4
f

(x)
g

(x)
= lim
x4
1
1
2

x
1
=
3
4
.
Hence by the LHopital Rule lim
x4
f(x)
g(x)
= lim
x4
x

x2
4x
=
3
4
.
Example 4.20. For a, b R, b = 0 lim
x1
sin(x
a
)
sin(x
b
)
=
a
b
.
Proof. Setting f(x) = sin(x
a
) and g(x) = sin(x
b
), we see that lim
x1
f(x) = lim
x1
g(x) = 0.
Moreover f

(x) = ax
a1
cos(x
a
) and g

(x) = bx
b1
cos(x
b
). Then g

does not vanish on


an open interval containing 1 and
lim
x1
f

(x)
g

(x)
= lim
x1
ax
a1
cos(x
a
)
bx
b1
cos(x
b
)
=
a
b
.
Hence by the LHopital Rule lim
x1
f(x)
g(x)
=
sin(x
a
)
sin(x
b
)
=
a
b
.
Theorem 4.18 is also known as the 0/0 LHopital Rule. The following theorem is the /
LHopital Rule.
Theorem 4.21. Let a < c < b and f, g : (a, b) R are dierentiable on (a, b)\{c}. Assume
also that g

(x) = 0 on (a, b) \ {c}, lim


xc
|f(x)| = lim
xc
|g(x)| = + and there exists the limit
lim
xc
f

(x)
g

(x)
= R. Then the limit lim
xc
f(x)
g(x)
does exist and equals .
Proof. Not examinable. The method (apply the Cauchy mean value theorem) is the same
as in the proof of Theorem 4.18, but the proof gets slightly more technical.
Remark 4.22. Theorems 4.18 and 4.21 work for one-sided limits as well. The results of
Theorems 4.18 and 4.21 remain true for limits at + or .
40
5 Riemann integral
The word integration is used in school calculus in two ways that, at rst sight, do not seem
to have much in common:
(a) given a function f, nd another function g whose derivative is f (such a g is called an
indenite integral or a primitive of f);
(b) given a function f (say, with positive values) over an interval [a, b], nd the area
enclosed by its graph, the x-axis, and the lines x = a and x = b (this is called the
denite integral of f over the interval).
It turns out that the two ideas are so closely connected (at least for simple functions)
that they are almost the same thing. School calculus is therefore free to concentrate on
learning various tricks for doing (a) in the case where f is given by simple formulae, and
(b) follows almost for free. For instance: Find the area under the graph of f(x) = x cos(x
2
)
for 0 x
_

2
. Solution: By some trick (change-of-variable in this case) we nd the
function g(x) =
1
2
sin(x
2
), whose derivative is f(x). Then the area required is
_

/2
0
f(x) dx = g
_
_
/2
_
g(0) =
1
2
sin
_
/2
_

1
2
sin(0) =
1
2
.
However, many perfectly reasonable functions are not the derivative of anything, and even
for those that are, there is often no routine formula for the indenite integral. One way to
deal with this problem is to look more carefully at the area under a graph idea and to
dene it in a rigorous way.
The idea is to split the interval [a, b], where the function f is dened, into small intervals
and approximate f from above and from below by functions taking constant values on each
of the small interval. In this way we approximate the area under the graph from above and
below by the areas of two histogram-like gures. The latter areas are nite sums of areas
of rectangles.
5.1 Denition of the Riemann Integral
Denition 5.1. A partition of an interval [a, b] is a nite set T of numbers in [a, b] that
includes both a and b. In other words we can write T = {t
0
, t
1
, . . . , t
n
}, where a = t
0
< t
1
<
. . . < t
n1
< t
n
= b. The points t
j
divide [a, b] up into subintervals
[t
0
, t
1
], [t
1
, t
2
], . . . , [t
n1
, t
n
].
Denition 5.2. We say that a partition T
1
of [a, b] is ner then a partition T
2
of [a, b] (or
T
2
is a renement of T
1
, or T
2
renes T
1
) if T
1
T
2
.
Example 5.3. Consider the partitions T
1
= {1, 0, 1}, T
2
= {1, 0, 1/2, 1} and T
3
=
{1, 1/2, 0, 1} of the interval [1, 1]. Then T
2
and T
3
are both ner than T
1
but neither
of T
2
, T
3
is ner than the other. It is easy to nd some partition that is ner than both T
2
and T
3
, for example: T
4
= T
2
T
3
= {1, 1/2, 0, 1/2, 1}.
41
Note that the following elementary properties are satised:
(a) For any two partitions T
1
, T
2
of [a, b], the partition T
3
= T
1
T
2
is a renement of each
of them.
(b) For any partition T = {a = t
0
, t
1
, t
2
, . . . , t
n
= b} of [a, b], with t
0
< t
1
< . . . < t
n
, we
have
n

k=1
(t
k
t
k1
) = b a.
Denition 5.4. Let f : [a, b] R be a bounded function and T = {t
0
, t
1
, . . . , t
n
} be a
partition of [a, b], whose elements are enumerated in the increasing order: a = t
0
< t
1
<
. . . < t
n1
< t
n
. For each j {1, . . . , n} denote M
j
(f, T) = sup
t
j1
st
j
f(s) and m
j
(f, T) =
inf
t
j1
st
j
f(s). Then the number
S(f, T) =
n

j=1
(t
j
t
j1
)M
j
(f, T)
is called the upper integral sum for f with respect to T and
S(f, T) =
n

j=1
(t
j
t
j1
)m
j
(f, T)
is called the lower integral sum for f with respect to T.
Example 5.5. Let f : [0, 10] R, f(x) = x and T = {0, 1, 2, 3, 4, 5, 6, 7, 8, 9, 10}. Clearly T
is a partition of [0, 10]. Since f is increasing M
j
(f, T) = sup
j1sj
f(s) = j, while m
j
(f, T) =
inf
j1sj
f(s) = j. Hence S(f, T) =
10

j=1
(j (j 1))j =
10

j=1
j = 55, while S(f, T) =
10

j=1
(j
(j 1))(j 1) =
10

j=1
(j 1) = 45.
The following lemma summarizes some basic properties of the upper and lower Riemann
integral sums.
Lemma 5.6. Let f : [a, b] R be a bounded function and T
1
, T be two partitions of [a, b]
such that T
1
is ner than T. Then
S(f, T) S(f, T
1
) S(f, T
1
) S(f, T).
Proof. Since M
j
(f, T
1
) m
j
(f, T
1
), we have S(f, T
1
) S(f, T
1
). Thus it remains to prove
that
S(f, T) S(f, T
1
) and S(f, T
1
) S(f, T).
It suces to prove these inequalities in the case when T
1
\ T is a singleton (=one-element
set). Indeed, assume that the required inequality is correct in this case. Since T
1
is nite,
so is T
1
\ T. That is, T
1
\ T = {s
1
, . . . , s
k
}. Let T
(0)
= T and T
(j)
= T {s
1
, . . . , s
j
} for
42
1 j k. Then each T
(j)
for 1 j k is a renement of T
(j1)
and T
(j)
\ T
(j1)
= {s
j
} is
a singleton. By our assumption
S(f, T
(j1)
) S(f, T
(j)
) and S(f, T
(j)
) S(f, T
(j1)
) for 1 j .
Since T
(k)
= T
1
, we have
S(f, T) = S(f, T
(0)
) S(f, T
(1)
) . . . S(f, T
(n)
) = S(f, T
1
)
and
S(f, T) = S(f, T
(0)
) S(f, T
(1)
) . . . S(f, T
(n)
) = S(f, T
1
).
Hence the required inequalities are satised.
Thus it remains to consider the case T
1
\ T = {s}. Let T = {t
0
, t
1
, . . . , t
n
} with a = t
0
<
t
1
< . . . < t
n
= b and k be such that s (t
k1
, t
k
). The terms in the sums dening S(f, T)
and S(f, T
1
) corresponding to the partition intervals [t
j1
, t
j
] with j < k or j > k are then
the same. Thus
S(f, T) S(f, T
1
) = (t
k
t
k1
) sup
t
k1
rt
k
f(r) (s t
k1
) sup
t
k1
ws
f(w) (t
k
s) sup
spt
k
f(p).
The trivial inequalities sup
spt
k
f(p) sup
t
k1
rt
k
f(r) and sup
t
k1
ws
f(w) sup
t
k1
rt
k
f(r) (the
supremum of a subset does not exceed the supremum of the entire set) imply that
(t
k
t
k1
) sup
t
k1
rt
k
f(r) (s t
k1
) sup
t
k1
ws
f(w) (t
k
s) sup
spt
k
f(p)
(t
k
t
k1
) sup
t
k1
rt
k
f(r) (s t
k1
) sup
t
k1
rt
k
f(r) (t
k
s) sup
t
k1
rt
k
f(r) = 0.
According to the last two displays S(f, T) S(f, T
1
) 0. That is S(f, T) S(f, T
1
).
Similarly we deal with the lower sums:
S(f, T) S(f, T
1
) = (t
k
t
k1
) inf
t
k1
rt
k
f(r) (s t
k1
) inf
t
k1
ws
f(w) (t
k
s) inf
spt
k
f(p).
The trivial inequalities inf
spt
k
f(p) inf
t
k1
rt
k
f(r) and inf
t
k1
ws
f(w) inf
t
k1
rt
k
f(r) (the
inmum of a subset does not exceed the inmum of the entire set) imply that
(t
k
t
k1
) inf
t
k1
rt
k
f(r) (s t
k1
) inf
t
k1
ws
f(w) (t
k
s) inf
spt
k
f(p)
(t
k
t
k1
) inf
t
k1
rt
k
f(r) (s t
k1
) inf
t
k1
rt
k
f(r) (t
k
s) inf
t
k1
rt
k
f(r) = 0.
According to the last two displays S(f, T) S(f, T
1
) 0. That is S(f, T) S(f, T
1
). The
proof is complete.
Corollary 5.7. For any two partitions T
1
and T
2
of [a, b] and any bounded function f :
[a, b] R, we have
S(f, T
1
) S(f, T
2
).
Proof. Let T
3
= T
1
T
2
. Then T
3
renes both T
1
and T
2
. By Lemma 5.6,
S(f, T
1
) S(f, T
3
) S(f, T
3
) S(f, T
2
)
as required.
43
Remark 5.8. Let M = M(f) = sup
asb
f(s) and m = m(f) = inf
asb
f(s) for a bounded
function f : [a, b] R. Then for any partition T of [a, b],
m(b a) S(f, T) S(f, T) M(b a).
Indeed, let T
0
= {a, b}. Then T renes T
0
and m(b a) = S(f, T
0
), S(f, T
0
) = M(b a). It
remains to apply Lemma 5.6.
Denition 5.9. Let f : [a, b] R be a bounded function. Since the set
{S(f, T) : T is a partition of [a, b]}
is bounded from below (by any particular lower integral sum), we can consider the number
J(f) = J(f, [a, b]) = inf
T
S(f, T).
It is called the upper Riemann integral of f on [a, b].
Similarly the set
{S(f, T) : T is a partition of [a, b]}
is bounded from above (by any particular upper integral sum) and we can consider the
number
J(f) = J(f, [a, b]) = inf
T
S(f, T).
It is called the lower Riemann integral of f on [a, b].
Since any lower integral sum of f does not exceed any upper integral sum of f (see
Corollary 5.7), we get
J(f, [a, b]) J(f, [a, b]).
Now we can dene Riemann integrability and Riemann integral.
Denition 5.10. A function f : [a, b] R is called Riemann integrable if it is bounded
and J(f, [a, b]) = J(f, [a, b]). In this case the number J(f, [a, b]) = J(f, [a, b]) is called the
Riemann integral of f on [a, b] and denoted
_
b
a
f(x) dx.
Example 5.11. A constant function f(x) = c R is Riemann integrable on any interval
[a, b] and
_
b
a
c dx = c(b a).
Proof. Let T = {t
0
, . . . , t
n
} be any partition of [a, b]. Since f is constant c, M
j
(f, T) =
m
j
(f, T) = c for each j. Hence
S(f, T) = S(f, T) =
n

j=1
(t
j
t
j1
)c = c
n

j=1
(t
j
t
j1
) = c(b a).
That is, J(f, [a, b]) = J(f, [a, b]) = c(b a). Thus f = c is integrable and
_
b
a
c dx =
c(b a).
44
Example 5.12. The Dirichlet function
f(x) =
_
1 if x Q,
0 if x / Q
is not Riemann integrable on any interval [a, b].
Proof. Let T = {t
0
, . . . , t
n
} be any partition of [a, b]. Since both Q and R \ Q are dense in
R, we see that M
j
(f, T) = 1 and m
j
(f, T) = 0 for each j. Hence
S(f, T) =
n

j=1
(t
j
t
j1
) 1 = b a
and
S(f, T) =
n

j=1
(t
j
t
j1
) 0 = 0.
Hence J(f, [a, b]) = 0 and J(f, [a, b]) = b a > 0. Thus f is not Riemann integrable.
5.2 Darbaux Criterion of Riemann Integrability
The following criterion is very similar to the Cauchy criterion of convergence of sequences
or series.
Theorem 5.13. Let f : [a, b] R be a bounded function. Then f is Riemann integrable if
and only if for each > 0 there exists a partition T of [a, b] such that
S(f, T) S(f, T) < .
Proof. First, assume that f is Riemann integrable and I =
_
b
a
f(x) dx. Then I = J(f, [a, b])
and I = J(f, [a, b]). By denitions of the upper and lower integrals, we can nd partitions
T
1
and T
2
of [a, b] such that S(f, T
1
) < I + /2 and S(f, T
2
) > I /2. Then S(f, T
1
)
S(f, T
2
) < . Let T = T
1
T
2
. Then T renes both T
1
and T
2
and therefore by Lemma 5.6,
S(f, T
1
) S(f, T) and S(f, T
1
) S(f, T). Thus the inequality S(f, T
1
) S(f, T
2
) <
implies that S(f, T) S(f, T) < as required.
Assume now that for each > 0 there is a partition T of [a, b] such that S(f, T)S(f, T) <
. Let be any positive number and T be such a partition of [a, b] that S(f, T)S(f, T) < .
Since
S(f, T) J(f, [a, b]) J(f, [a, b]) S(f, T),
we see that 0 J(f, [a, b]) J(f, [a, b]) < . Since > 0 is arbitrary, J(f, [a, b]) = J(f, [a, b])
and therefore f is integrable as required.
We skip the proof of the following result. Do it as an exercise. For a partition T =
{t
0
, . . . , t
n
} of [a, b], the number d(T) = min
1jn
(t
j
t
j1
) is called the diameter of T.
Proposition 5.14. Let f : [a, b] R. Then the limits lim
d(T)0
S(f, T) and lim
d(T)0
S(f, T) do
exist and equal J(f, [a, b]) and J(f, [a, b]) respectively.
45
That is, instead of supremum/inmum denitions of upper and lower integrals we could
use the limit ones.
Theorem 5.15. If f : [a, b] R is continuous, then f is Riemann integrable.
Proof. We shall apply the Darbaux Criterion (Theorem 5.13). Let > 0. Since f is a
continuous function on a bounded closed interval, f is bounded and uniformly continuous,
see Theorem 2.7. Since f is uniformly continuous, there is > 0 such that
|f(x) f(y)| <

b a
whenever |x y| < .
Now take any partition T = {t
0
, . . . , t
n
} of [a, b] with d(T) < . According to the last
display M
j
(f, T) m
j
(f, T) <

ba
for each j {1, . . . , n}. Multiplying these inequalities by
(t
j
t
j1
) and adding up, we get
S(f, T) S(f, T) =
n

j=1
(t
j
t
j1
)(M
j
(f, T) m
j
(f, T))

b a
n

j=1
(t
j
t
j1
) = .
It remains to apply Theorem 5.13 to conclude that f is Riemann integrable.
Theorem 5.16. If f : [a, b] R is monotone (=increasing or decreasing), then f is Rie-
mann integrable.
Proof. If f is a constant function, we already know that f is integrable. Assume now that f
is not constant. Since f is monotone M = |f(b) f(a)| > 0. Let > 0. Take any partition
T of [a, b] with d(T) <

M
. Using monotonicity of f, we observe that M
j
(f, T) m
j
(f, T) =
|f(t
j
) f(t
j1
)| for every j. Thus
S(f, T) S(f, T) =
n

j=1
(t
j
t
j1
)(M
j
(f, T) m
j
(f, T)) <

M
n

j=1
|f(t
j
) t
j1
|.
If f is increasing, then |f(t
j
) t
j1
| = f(t
j
) t
j1
for every j. If f is decreasing, then
|f(t
j
)t
j1
| = (f(t
j
)t
j1
) for every j. Thus in any case
n

j=1
|f(t
j
)t
j1
| = |f(t
n
)f(t
0
)| =
|f(b) f(a)| = M. By the above display, S(f, T) S(f, T) <

M
M = . It remains to apply
Theorem 5.13 to conclude that f is Riemann integrable.
5.3 Properties of the Riemann integral
Theorem 5.17. If a bounded function f : [a, b] R is Riemann integrable on [a, b], then it
is Riemann integrable on any [c, d] [a, b].
Proof. Let > 0. By Theorem 5.13, there is a partition T of [a, b] such that S(f, T)
S(f, T) < . Let T

= T {c, d}. Since T

is a renement of T, Lemma 5.6 implies that


S(f, T

) S(f, T

) < . Let T

be the partition of [c, d] given by T

= T

[c, d] (it is a
partition of [c, d] since c, d T

). From the denitions of upper and lower integral sums, we


immediately get
S(f, T

) S(f, T

) S(f, T

) S(f, T

) < .
Since > 0 is arbitrary, Theorem 5.13 provides integrability of f on [c, d].
46
Theorem 5.18. Let a < c < b and f : [a, b] R is Riemann integrable on [a, c] and on
[b, c]. Then it is Riemann integrable on [a, b] and
_
b
a
f(x) dx =
_
c
a
f(x) dx +
_
b
c
f(x) dx.
Proof. If f is integrable on [a, b], then by Theorem 5.17, f is integrable on [a, c] and on [b, c].
Assume now that f is integrable on [a, c] and on [b, c]. Let > 0. By Theorem 5.13,
there are a partition T
1
= {t
0
, t
1
, . . . , t
k
} with a = t
0
< t
1
< . . . < t
k
= c of [a, c] and
a partition T
2
= {t
k
, t
k+1
, . . . , t
n
} with c = t
k
< t
k+1
< . . . < t
n
= b of [c, b] such that
S(f, T
1
) S(f, T
1
) < /2 and S(f, T
2
) S(f, T
2
) < /2. Then T = T
1
T
2
= {t
0
, t
1
, . . . , t
n
}
is a partition of [a, b]. From the denitions of the upper and lower integral sums, we have
S(f, T) = S(f, T
1
) + S(f, T
2
) and S(f, T) = S(f, T
1
) + S(f, T
2
). Let J
1
=
_
c
a
f(x) dx and
J
2
=
_
b
c
f(x) dx. By the denition of the Riemann integral, S(f, T
1
) J
1
S(f, T
1
)
and S(f, T
2
) J
2
S(f, T
2
). Adding these two inequalities and taking into account the
equalities S(f, T) = S(f, T
1
) + S(f, T
2
) and S(f, T) = S(f, T
1
) + S(f, T
2
), we get
S(f, T) J
1
+ J
2
S(f, T).
Adding the inequalities S(f, T
1
) S(f, T
1
) < /2 and S(f, T
2
) S(f, T
2
) < /2 and using
the equalities S(f, T) = S(f, T
1
) + S(f, T
2
) and S(f, T) = S(f, T
1
) + S(f, T
2
), we get
S(f, T) S(f, T) < .
From the last two displays, we have
J
1
+ J
2
< S(f, T) S(f, T) < J
1
+ J
2
+ .
Hence J
1
+ J
2
J(f, [a, b]) J(f, [a, b]) J
1
+ J
2
+ . Since > 0 is arbitrary,
J(f, [a, b]) = J(f, [a, b]) = J
1
+ J
2
. That is, f is integrable on [a, b] and
_
b
a
f(x) dx =
_
c
a
f(x) dx +
_
b
c
f(x) dx.
The following two theorems together provide the property of the Riemann integral, known
as its linearity.
Theorem 5.19. Suppose that f and g are Riemann integrable over [a, b]. Then so is f +g,
and
_
b
a
(f + g)(x)a, dx =
_
b
a
f(x) dx +
_
b
a
g(x) dx.
Proof. Let J
1
=
_
b
a
f(x) dx and J
2
=
_
b
a
g(x) dx. Take > 0. By denition of the Riemann
integral (we use denitions of inf and sup, through which the Riemann integral is dened),
there are partitions T
1
, T
2
, T
3
and T
4
of [a, b] such that S(f, T
1
) < J
1
+

2
, S(f, T
2
) > J
1


2
,
S(g, T
3
) < J
2
+

2
, S(g, T
4
) > J
2


2
. Now take T = T
1
T
2
T
3
T
4
. Since T is ner than
each of T
j
with 1 j 4, we can use the above inequalities and Lemma 5.6, to see that
J
1


2
< S(f, T) S(f, T) < J
1
+

2
and
J
2


2
< S(g, T) S(g, T) < J
2
+

2
.
47
Adding these inequalities we get
J
1
+ J
2
< S(f, T) + S(g, T) S(f, T) + S(g, T) < J
1
+ J
2
+ . (5.1)
On the other hand, using the denitions of inf and sup, we see that m
j
(f +g, T) m
j
(f, T)+
m
j
(g, T) and M
j
(f +g, T) M
j
(f, T) +M
j
(g, T). Multiplying these inequalities by t
j
t
j1
and adding them up, we get S(f + g, T) S(f, T) + S(g, T) and S(f + g, T) S(f, T) +
S(g, T). These two inequalities together with (5.1) give
J
1
+ J
2
< S(f + g, T) S(f + g, T) < J
1
+ J
2
+ .
By denitions of the upper and lower integrals, we get
J
1
+ J
2
< J(f + g) J(f + g) < J
1
+ J
2
+ .
Since > 0 is arbitrary, we have J(f + g) = J(f + g) = J
1
+ J
2
. That is, f + g is Riemann
integrable and
_
b
a
(f + g)(x)a, dx = J
1
+ J
2
, as required.
Theorem 5.20. Suppose that f is Riemann integrable over [a, b], and c R. Then cf is
also Riemann integrable and
_
b
a
cf(x) dx = c
_
b
a
f(x) dx.
Proof. Let J =
_
b
a
f(x) dx. The case c = 0 is trivial (cf is the constant 0 function). Assume
now that c = 0. Take > 0. By denition of the Riemann integral (we use denitions of inf
and sup, through which the Riemann integral is dened), there are partitions T
1
and T
2
of
[a, b] such that S(f, T
1
) < J +

|c|
and S(f, T
2
) > J
1


|c|
. Now take T = T
1
T
2
. Since T is
ner than T
1
and than T
2
, we can use the above inequalities and Lemma 5.6, to see that
J

|c|
< S(f, T) S(f, T) < J +

|c|
.
If c > 0, we, using the denitions of inf and sup, see that m
j
(cf, T) = cm
j
(f, T) and
M
j
(f, T) = cM
j
(f, T). Similarly, for c < 0, we see that m
j
(cf, T) = cM
j
(f, T) and
M
j
(f, T) = cm
j
(f, T). Multiplying these inequalities by t
j
t
j1
and adding them up,
we get S(cf, T) = cS(f, T) and S(cf, T) = cS(f, T) if c > 0 and S(cf, T) = cS(f, T) and
S(cf, T) = cS(f, T) if c < 0. In any case, these inequalities together with the last display
give
cJ < S(cf, T) S(cf, T) < J + .
By denitions of the upper and lower integrals, we get
J < J(cf) J(cf) < J + .
Since > 0 is arbitrary, we have J(cf) = J(cf) = cJ. That is, cf is Riemann integrable
and
_
b
a
(cf)(x)a, dx = cJ, as required.
It happens that if two functions dier only at nitely many points, then their integrability
status and the value of the integral (if integrable) is the same. This is formalized in the next
theorem.
48
Theorem 5.21. Let f, g : [a, b] R be such that the set A = {x [a, b] : f(x) = g(x)} is
nite. Then Riemann integrability of f implies Riemann integrability of g and
_
b
a
f(x) dx =
_
b
a
g(x) dx.
Proof. Let h = g f. Then h(x) = 0 for every x [a, b] \ A. Since A is nite, we can write
A = {s
1
, . . . , s
r
}. Let also M = max{|h(s
1
)|, . . . , |h(s
r
)|}. Fix > 0 and let =

2Mr
. Take
any partition T of [a, b] with d(T) < . Since h vanishes everywhere except for the points s
j
,
M
j
(h, T) = m
j
(h, T) = 0 if [t
j1
, t
j
] contains none of the points s
k
.
Since M h(x) M for every x [a, b], we have
M m
j
(h, T) M
j
(h, T) M if [t
j1
, t
j
] contains an s
k
.
Since each s
k
is contained in at most two of the intervals [t
j1
, t
j
] (normally s
k
is contained
in exactly one interval dened by the partition, only in the case s
k
= t
j
with j < n, s
k
is
contained in [t
j1
, t
j
] and in [t
j
, t
j+1
]), the equality M
j
(h, T) = m
j
(h, T) = 0 holds for all j
except for at most 2r of them. Thus
S(f, T) =

j:[t
j1
,t
j
] contains an s
k
(t
j
t
j1
)M
j
(h, T) 2rd(T)M < 2rM =
and similarly
S(f, T) =

j:[t
j1
,t
j
] contains an s
k
(t
j
t
j1
)m
j
(h, T) 2rd(T)M > 2rM = .
By denition of the upper and lower integrals, we have
J(h, [a, b]) J(h, [a, b]) .
Since > 0 is arbitrary, J(h) = J(h) = 0. Hence h is Riemann integrable and
_
b
a
h(x) dx = 0.
Assume now that f is Riemann integrable. By Theorem 5.19, g = f + h is Riemann
integrable and
_
b
a
g(x) dx =
_
b
a
f(x) dx +
_
b
a
h(x) dx =
_
b
a
f(x) dx, as required.
Theorem 5.22. Let f : [a, b] [c, d] be a Riemann integrable function and g : [c, d] R
be a continuous function. Then the composition g f is Riemann integrable on [a, b].
Proof. Our main instruments are the Darboux Criterion and uniform continuity co contin-
uous functions on a bounded closed interval. First, since g : [c, d] R is continuous, by
Theorem 0.18, g is bounded. That is, there exists M > 0 such that |g(x)| M for every
x [c, d].
Take , > 0. Since f is Riemann integrable, the Darboux Criterion (Theorem 5.13)
implies that there is a partition T = {t
0
, . . . , t
n
} with a = t
0
< t
1
< . . . < t
n
= b of [a, b]
such that S(f, T) S(f, T) < . By Theorem 2.7, g is uniformly continuous. Hence, there
is = () > 0 such that |g(x) g(y)| whenever x, y [c, d], |x y| . If j is such
that M
j
(f, T) m
j
(f, T) , then |f(t) f(s)| and therefore |g(f(t)) g(f(s))|
49
for every t, s [t
j1
, t
j
]. For such a j, we then have M
j
(g f, T) m
j
(g f, T) . It
follows that

j:M
j
(f,T)m
j
(f,T)
(t
j
t
j1
)(M
j
(g f, T) m
j
(g f, T))


j:M
j
(f,T)m
j
(f,T)
(t
j
t
j1
)
n

j=1
(t
j
t
j1
) = (b a).
If j is such that M
j
(f, T)m
j
(f, T) > , we have 1 <
M
j
(f,T)m
j
(f,T)

. Since |g| M, we have


|g(f(t)) g(f(s))| 2M for every t, s [t
j1
, t
j
] and therefore M
j
(g f, T) m
j
(g f, T)
2M <
2M(M
j
(f,T)m
j
(f,T))

. Hence

j:M
j
(f,T)m
j
(f,T)
(t
j
t
j1
)(M
j
(g f, T) m
j
(g f, T))


j:M
j
(f,T)m
j
(f,T)
2M(M
j
(f, T) m
j
(f, T))(t
j
t
j1
)

2M

(S(f, T) S(f, T)) <


2M

.
Summing up the estimates in the above two displays, we get
S(g f, T) S(g f, T) =
n

j=1
(t
j
t
j1
)(M
j
(g f, T) m
j
(g f, T)) < (b a) +
2M

.
This estimates hold for any choice of positive and . Now x > 0. Take =

2(ba)
and =
()
4M
. Then the above display gives S(g f, T) S(g f, T) <

2
+

2
= . It
remains to apply the Darboux Criterion (Theorem 5.13) to conclude that g f is Riemann
integrable.
Theorem 5.23. Suppose that f is Riemann integrable over [a, b]. Then so is f
2
.
Proof. Apply Theorem 5.22 with g(x) = x
2
.
Theorem 5.24. If f and g are Riemann integrable over [a, b] then so is fg.
Proof. Note rst that fg =
1
4
{(f + g)
2
(f g)
2
}. Now the right-hand side is Riemann
integrable by Theorems 5.19, 5.20 and 5.22.
Theorem 5.25. Suppose that f is Riemann integrable over [a, b]. Then so is |f|.
Proof. Apply Theorem 5.22 with g(x) = |x|.
Theorem 5.26. Let f : [a, b] R be a non-negative Riemann integrable function. Then
_
b
a
f(x) dx 0.
Proof. All upper and all lower integral sums of a non-negative function are non-negative.
Hence so are their upper and lower integrals. The result follows.
50
Theorem 5.27. If f and g are Riemann integrable over [a, b] and f(x) g(x) for all
x [a, b], then
_
b
a
f(x) dx
_
b
a
g(x) dx.
Proof. Apply Theorem 5.26 to g f and use Theorems 5.19, 5.20.
Corollary 5.28. If f is Riemann integrable over [a, b], then

_
b
a
f(x) dx

_
b
a
|f(x)| dx.
Proof. By Theorem 5.25, |f| is integrable. Since |f(x)| f(x) |f(x)| for each x [a, b],
Theorem 5.27 gives

_
b
a
|f(x)| dx
_
b
a
f(x) dx
_
b
a
|f(x)| dx
and the required inequality follows.
The following result is called the Mean Value Theorem for Integrals.
Theorem 5.29. Suppose that f is continuous on [a, b]. Then there is c (a, b) such that
_
b
a
f(x) dx = (b a)f(c).
Proof. By Theorem 0.18, there are , [a, b] such that m = f() f(x) f() = M
for each x [a, b] (m and M are the maximal and the minimal values of f respectively).
By Theorem 5.27, m(b a) =
_
b
a
mdx
_
b
a
f(x) dx
_
b
a
M dx = M(b a). Hence the
number
1
ba
_
b
a
f(x) dx lies between m = f() and M = f(). By the intermediate value
theorem (Theorem 0.17), there is c between and (and therefore in [a, b]) such that
f(c) =
1
ba
_
b
a
f(x) dx. The required equality follows.
Remark 5.30. When b < a we interpret
_
a
b
f(x) dx to mean
_
b
a
f(x) dx; also
_
a
a
f(x) dx
means 0. With these conventions, Theorem 5.29 becomes valid no matter what order a and
b are in (c must be between a and b).
Theorem 5.31. Given a Riemann integrable function f : [a, b] R, the function F :
[a, b] R dened by the formula
F(x) =
_
x
a
f(t) dt for each x [a, b].
is continuous.
Proof. Since f is Riemann integrable, it is bounded. Hence there is M > 0 such that
|f(x)| M for each x [a, b]. Let x, h R be such that x, x +h [a, b]. By Theorem 5.18,
F(x + h) F(x) =
_
x+h
a
f(t) dt
_
x
a
f(t) dt =
_
x+h
x
f(t) dt.
By Theorem 5.29, there is w
x,h
between x and x + h such that
F(x + h) F(x) =
_
x+h
x
f(t) dt = hf(w
x,h
).
Since |f(w
x,h
)| M, we have |F(x + h) F(x)| M|h|. By the Sandwich rule F(x + h)
F(x) 0 as h 0. Thus F(x + h) F(x) as h 0. That is, F is continuous at x. Since
x [a, b] is arbitrary, F is continuous on [a, b].
51
Theorem 5.32. Let f : [a, b] R be a continuous function and F : [a, b] R be dened by
the formula F(x) =
_
x
a
f(x) dx. Then F is dierentiable, with derivative f, on (a, b).
Proof. Fix c (a, b) and let h R be such that h = 0 and c + h [a, b]. Then F(c +
h) F(c) =
_
c+h
c
f(x) dx. By Theorem 5.29, there is d(h) between c and c + h such that
F(c + h) F(c) =
_
c+h
c
f(x) dx = ((c + h) c)f(d(h)) = hf(d(h)). Hence
F(c+h)F(c)
h
=
f(d(h)). Since d(h) lies between c and c + h and c + h c as h 0, we have by the
sandwich rule that d(h) c as h 0. Since f is continuous, f(d(h)) f(c) as h 0.
Since
F(c+h)F(c)
h
= f(d(h)), we have lim
h0
F(c+h)F(c)
h
= f(c). By denition, F is dierentiable
at c and F

(c) = f(c). Since c is an arbitrary point of (a, b), the result follows.
The following theorem is known as the fundamental theorem of calculus.
Theorem 5.33. Let f : [a, b] R be a continuous function and suppose that there is a
continuous function G : [a, b] R such that G is dierentiable on (a, b) and G

(x) = f(x)
for each x (a, b). Then
_
b
a
f = G(b) G(a).
Proof. Consider F : [a, b] R dened by the formula F(x) =
_
x
a
f(x) dx. By Theorems 5.32
and 5.31, F is continuous on [a, b], dierentiable on (a, b) and F

= f. Hence the function


F G is continuous on [a, b] and has zero derivative everywhere on (a, b). Thus it is constant:
G(x) F(x) = c for all x [a, b], where c R does not depend on x. Hence G(b) G(a) =
(F(b) + c) (F(a) + c) = F(b) F(a) =
_
b
a
f(x) dx (the latter equality holds by denition
of F) as required.
Example 5.34. If 0 < a < b and f(x) =
1
x
on [a, b], then f is Riemann integrable over [a, b]
and
_
b
a
f(x) dx = ln(b/a).
Proof. First, f is R-integrable because it is continuous. Second, we notice that
d
dx
(ln(x)) =
1
x
= f(x) on (a, b), so by Theorem 5.33,
_
b
a
f(x) dx = ln(b) ln(a) = ln(b/a).
We end with justication of the two results from school calculus. Namely, the substitution
and the integration by parts formula. The next theorem deals with substitution (=change
of variables).
Theorem 5.35. Let g : [a, b] [c, d] be a function with continuous derivative and f :
[c, d] R be continuous. Then
_
g(b)
g(a)
f(t) dt =
_
b
a
(f g)(x)g

(x) dx.
Proof. The functions under both integrals in the required formula are continuous and there-
fore Riemann integrable. It remains to prove the equality. Let F : [a, b] R dened
by the formula F(x) =
_
x
a
f(x) dx. By Theorems 5.32 and 5.31, F is continuous on [a, b],
dierentiable on (a, b) and F

= f. By Theorem 5.33,
_
g(b)
g(a)
f(t) dt = F(g(b)) F(g(a)).
52
On the other hand using the chain rule (=derivative of composition formula) (F g)

(x) =
F

(g(x))g

(x) = f(g(x))g

(x) for every x (a, b). By Theorem 5.33,


_
b
a
(f g)(x)g

(x) dx = F g(b) F g(a) = F(g(b)) F(g(a)).


The required equality follows from the above two displays.
The next theorem is known as the integration by parts formula.
Theorem 5.36. Assume that f, g : [a, b] R both have continuous derivatives on [a, b].
Then fg

and gf

are Riemann integrable over [a, b], and


_
b
a
f(x)g

(x) = f(b)g(b) f(a)g(a)


_
b
a
g(x)f

(x).
Proof. The functions under both integrals in the required formula are continuous and there-
fore Riemann integrable. It remains to prove the required equality. By the Leibniz rule (the
derivative of the product formula) (fg)

= fg

+ gf

. By Theorem 5.33,
_
b
a
(f(x)g

(x) + g(x)f

(x)) dx = (fg)(b) (fg)(a) = f(b)g(b) f(a)g(a).


In view of the linearity of the Riemann integral the above equality is equivalent to the
required one.
53

Você também pode gostar